51. After the birth of twins, the post parturient woman developed a massive hemorrhage from the natural birth canal. The placenta and birth canal are intact. The uterine fundus is located above the navel, the uterus is soft to palpation and does not respond to the administration of uterotonics. What is the most likely cause of the bleeding in this case?
A. Damage to the uterine cervix
B. Uterine rupture
C. Retained fragment of the placenta
D. Uterine atony
E. Uterine hypotonia
Correct Answer: D. Uterine atony
Explanation:
Uterine atony is the most common cause of postpartum hemorrhage (PPH), especially in cases of multiple gestations (e.g., twins), where the uterus becomes overdistended and fails to contract effectively after delivery.
- Key clues in this case:
- Massive postpartum hemorrhage despite an intact placenta and birth canal
- Soft, enlarged uterus above the navel (indicating poor contraction)
- No response to uterotonics (suggesting severe atony)
In a normal postpartum state, uterine contractions compress the spiral arteries and prevent excessive bleeding. When the uterus fails to contract (atony), there is uncontrolled blood loss from the myometrial blood vessels.
Why Other Options Are Incorrect:
- A. Damage to the uterine cervix – Incorrect.
- Cervical tears cause localized bleeding but do not lead to a soft, enlarged uterus.
- Examination usually reveals visible lacerations, and bleeding persists even when the uterus contracts properly.
- B. Uterine rupture – Incorrect.
- Uterine rupture typically presents with sudden, severe abdominal pain, fetal distress, and signs of hypovolemic shock.
- In this case, there is no mention of rupture risk factors (e.g., previous C-section, traumatic labor).
- The uterus remains intact, making rupture unlikely.
- C. Retained fragment of the placenta – Incorrect.
- Retained placental tissue can cause delayed postpartum hemorrhage, but in this case, the placenta is intact.
- The uterus would typically be firm but irregularly contracted rather than diffusely soft and unresponsive.
- E. Uterine hypotonia – Incorrect.
- Uterine hypotonia refers to reduced uterine tone but some degree of contraction, whereas atony refers to complete loss of tone, leading to massive hemorrhage.
- In this case, the uterus is completely soft and unresponsive to uterotonics, suggesting true atony rather than mild hypotonia.
Conclusion:
This case describes classic postpartum uterine atony, a leading cause of postpartum hemorrhage, especially after multiple gestation deliveries. Immediate uterine massage, uterotonic drugs (oxytocin, ergometrine, misoprostol), and possibly surgical intervention (e.g., uterine compression sutures, hysterectomy) are required to control the bleeding.
52. A 2-day-old baby presents with melena and vomiting with blood. According to the patient’s medical history, the child was born with the body weight of 3400 g, the body length of 52 cm, and the Apgar score of 8. The mother has epilepsy and for its treatment she was taking anticonvulsants duri- ng her pregnancy. Complete blood count: erythrocytes-2.2 1012/L, hemoglobin- 112 g/L, hematocrit 0.38, platelets-185-109/L. What is the most likely diagnosis in this case?
A. Hemolytic disease of the newborn
B. Perinatal infection
C. Congenital aplastic anemia
D. Transimmune thrombocytopenic purpura
E. Hemorrhagic disease of the newborn
Correct Answer: E. Hemorrhagic disease of the newborn
Explanation:
This newborn presents with melena (bloody stools) and hematemesis (vomiting blood) on the second day of life, which is characteristic of hemorrhagic disease of the newborn (HDN), a condition caused by vitamin K deficiency leading to impaired coagulation.
- Key clues in this case:
- Newborn (2 days old) with bleeding symptoms (melena, hematemesis)
- Mother was taking anticonvulsants (e.g., phenytoin, phenobarbital, carbamazepine), which interfere with vitamin K metabolism
- Low erythrocyte count and moderate anemia, likely due to blood loss rather than hemolysis or infection
Pathophysiology:
Vitamin K is essential for the synthesis of coagulation factors II, VII, IX, and X. Newborns are naturally vitamin K-deficient due to:
- Low placental transfer of vitamin K
- Sterile gut at birth (lack of vitamin K-producing bacteria)
- Lack of vitamin K in breast milk
The condition is worsened if the mother took anticonvulsants, which impair fetal vitamin K metabolism, increasing the risk of early-onset hemorrhagic disease of the newborn (which typically occurs in the first 1–3 days after birth).
Why Other Options Are Incorrect:
- A. Hemolytic disease of the newborn – Incorrect.
- HDN results from Rh or ABO incompatibility and presents with jaundice, anemia, and reticulocytosis, not melena or hematemesis.
- This baby has no history of blood type incompatibility.
- B. Perinatal infection – Incorrect.
- Infections such as sepsis or TORCH infections can cause thrombocytopenia and petechiae, but they rarely cause isolated gastrointestinal bleeding in a well-appearing newborn.
- The baby’s Apgar score was good, and there are no signs of systemic infection (fever, lethargy, respiratory distress).
- C. Congenital aplastic anemia – Incorrect.
- Aplastic anemia (e.g., Fanconi anemia) presents with pancytopenia, congenital anomalies, and progressive bone marrow failure, not isolated gastrointestinal bleeding.
- This newborn’s platelet count is normal, which makes aplastic anemia unlikely.
- D. Transimmune thrombocytopenic purpura – Incorrect.
- This condition occurs when maternal antibodies cross the placenta and attack fetal platelets, leading to low platelet counts (<100 × 10⁹/L) and petechiae or purpura.
- This newborn has a normal platelet count (185 × 10⁹/L), making thrombocytopenia unlikely.
Conclusion:
This case is classic for hemorrhagic disease of the newborn (HDN) due to vitamin K deficiency, worsened by maternal anticonvulsant use. Early administration of intramuscular vitamin K at birth prevents this condition. Treatment includes vitamin K administration and, if needed, fresh frozen plasma for severe bleeding.
53. A 56-year-old patient complains of unbearable pain behind her sternum that does not stop even after taking three tablets of nitroglycerin. She has history of essential hypertension since the age of 45, ischemic heart disease, and frequent stresses at work. Objectively, the following is observed: noticeable skin pallor, acrocyanosis, cold sticky sweat, respiratory rate 24/min, inspiratory dyspnea, heart rate 118/min, the pulse is rapid, of poor volume, blood pressure – 90/50 mm Hg. The left border of the relative cardi- ac dullness is shifted outwards. Auscultation detects weakened heart sounds at the apex, tachycardia, and gallop rhythm. ECG registers QS waves in leads I, aVL, V4-V6. What is the most likely diagnosis in this case?
A. Pneumothorax
B. Pulmonary embolism
C. Myocardial infarction
D. Cardiogenic shock
E. Acute vascular insufficiency
Correct Answer: C. Myocardial infarction (MI
Explanation:
This 56-year-old patient presents with severe, persistent chest pain unrelieved by nitroglycerin, which is highly suggestive of an acute myocardial infarction (MI). The presence of hemodynamic instability (hypotension, tachycardia, cold sweat, acrocyanosis, and gallop rhythm) further supports the diagnosis of a large infarct with significant myocardial dysfunction.
Key diagnostic clues in this case:
- “Unbearable pain behind the sternum” that persists despite nitroglycerin (classic MI symptom)
- History of hypertension and ischemic heart disease (major risk factors for MI)
- Pallor, acrocyanosis, cold sticky sweat (signs of sympathetic activation and hypoperfusion)
- Hypotension (90/50 mmHg) and tachycardia (118/min) (suggestive of cardiogenic shock secondary to a large infarct)
- ECG findings: QS waves in leads I, aVL, V4–V6
- QS waves indicate transmural necrosis (a sign of a previous or evolving infarction)
- I, aVL (high lateral leads) and V4-V6 (anterolateral leads) suggest a large anterolateral MI
- The leftward shift of cardiac dullness suggests left ventricular dilation or dysfunction
Why Other Options Are Incorrect:
- A. Pneumothorax – Incorrect.
- Pneumothorax presents with sudden pleuritic chest pain and dyspnea, but it does not cause substernal pain, ST changes on ECG, or a gallop rhythm.
- Breath sounds would be asymmetrically decreased, which is not mentioned in this case.
- B. Pulmonary embolism (PE) – Incorrect.
- PE causes sudden onset pleuritic chest pain, dyspnea, tachypnea, and possible hemoptysis, but it does not typically cause a gallop rhythm or ST-segment changes suggestive of infarction.
- ECG in PE may show sinus tachycardia, S1Q3T3 pattern, or right heart strain, but not QS waves in anterolateral leads.
- D. Cardiogenic shock – Partially correct but incomplete.
- The patient is experiencing cardiogenic shock due to an extensive myocardial infarction.
- However, cardiogenic shock is a complication of MI, not a standalone primary diagnosis.
- E. Acute vascular insufficiency – Incorrect.
- This is a nonspecific term and does not accurately describe the underlying pathology in this case.
- The symptoms are best explained by an acute myocardial infarction leading to cardiogenic shock.
Conclusion:
The persistent severe chest pain, hemodynamic instability, and ECG findings (QS waves in anterolateral leads) confirm the diagnosis of an acute extensive anterolateral myocardial infarction. Immediate management should include oxygen, aspirin, P2Y12 inhibitors (clopidogrel/ticagrelor), heparin, morphine (if needed), and urgent revascularization with PCI or thrombolysis.
54 The effect of various risk factors on Exacerbation of peptic ulcer disease of the stomach was studied. Among other factors, the seasonality of this disease was studied. What group of risk factors does it belong to?
A. Exogenous uncontrollable
B. Exogenous controllable
C. Endogenous uncontrollable
D. Social controllable
E. Endogenous controllable
Correct Answer: A. Exogenous uncontrollable
Explanation:
Risk factors for peptic ulcer disease (PUD) exacerbation can be classified into exogenous (external) and endogenous (internal) factors, as well as controllable (modifiable) and uncontrollable (non-modifiable) factors.
- Seasonality refers to the observation that PUD exacerbations often occur in specific seasons (e.g., spring and autumn), possibly due to changes in diet, stress levels, and environmental factors.
- Since seasonality is an external (exogenous) factor that cannot be controlled, it falls under the category of exogenous uncontrollable risk factors.
Why Other Options Are Incorrect:
- B. Exogenous controllable – Incorrect.
- Controllable exogenous factors include smoking, alcohol use, NSAID use, diet, and stress—all of which can be modified.
- Seasonality is not controllable, so this category is incorrect.
- C. Endogenous uncontrollable – Incorrect.
- Endogenous factors are internal, such as genetic predisposition, blood type, and age.
- Seasonality is an external factor, so it does not fit here.
- D. Social controllable – Incorrect.
- Social factors include lifestyle, socioeconomic status, and diet, which can be modified.
- Seasonal changes in ulcer exacerbation are not directly controlled by social factors.
- E. Endogenous controllable – Incorrect.
- Controllable endogenous factors include H. pylori infection, gastric acid secretion, and medication use.
- Seasonality is not an internal (endogenous) factor, so this option is incorrect.
Conclusion:
Seasonality of peptic ulcer disease exacerbations is an external (exogenous) factor that cannot be controlled, making it an exogenous uncontrollable risk factor.
- A 40-year-old woman complains of white watery discharge and periodic contact- provoked bleeding from the vagina, observed over the last three months. She has history of childbirth complicated by cervical rupture. Examination of the cervix in the mirrors shows that the cervix is hypertrophied and has ulcerated areas that are easily injured and start bleeding when touched. What is the most likely diagnosis in this case?
A. Endometriosis
B. Cervical cancer
C. Cervical erosion
D. Ectropion
E. Cervical pregnancy
Correct Answer: B. Cervical cancer
Explanation:
This 40-year-old woman presents with white watery discharge and contact bleeding, which are classic signs of cervical cancer. The history of previous cervical trauma (cervical rupture during childbirth) increases the risk, as chronic inflammation and HPV infection can lead to malignant transformation.
Key diagnostic clues in this case:
- White watery discharge – Suggestive of necrosis or infection in a malignant lesion.
- Contact bleeding – A hallmark symptom of cervical cancer due to fragile tumor vasculature.
- Ulcerated, hypertrophied cervix that bleeds on touch – Strongly indicates an advanced neoplastic process.
Why Other Options Are Incorrect:
- A. Endometriosis – Incorrect.
- Endometriosis usually causes cyclical pelvic pain, dysmenorrhea, dyspareunia, and infertility.
- It does not present with persistent watery discharge and contact bleeding.
- C. Cervical erosion – Incorrect.
- Cervical erosion (ectopy) is a benign condition where columnar epithelium extends from the endocervix to the ectocervix.
- It can cause increased mucus production but does not typically present with ulceration, hypertrophy, or contact bleeding.
- D. Ectropion – Incorrect.
- Ectropion is a benign eversion of the endocervical mucosa seen in young women, pregnancy, or those on oral contraceptives.
- It does not cause ulcerated lesions or significant contact bleeding.
- E. Cervical pregnancy – Incorrect.
- Cervical pregnancy is a rare ectopic pregnancy implanted in the cervix, causing severe first-trimester bleeding.
- There is no mention of pregnancy in this case, and the symptoms have been progressing over months, making malignancy more likely.
Conclusion:
The persistent white discharge, contact bleeding, and ulcerated cervix strongly suggest cervical cancer. Further evaluation should include colposcopy, cervical biopsy, and HPV testing to confirm the diagnosis.
56. A 42-year-old woman lost her consciousness after physical overexertion. Her blood pressure dropped to 40/20 mm Hg. She has hi- story of long-term (5 years) use of glucocorticoids, because she has bronchial asthma. She has not been taking glucocorticoids for the last 4 days. Objectively, she is inert, her skin is of normal color and normal moisture level. Auscultattion detects muffled heart sounds, heart rate 100/min, of poor volume, rhythmic. Blood test: glucose 3.0 mmol/L, sodium 117 mmol/L, potassium 6.0 mmol/L. What is the most likely diagnosis in this case?
A. Status asthmaticus
B. Hypovolemic shock
C. Cardiogenic shock
D. Acute adrenal insufficiency
E. Hypoglycemic coma
Correct Answer: D. Acute adrenal insufficiency
Explanation:
This 42-year-old woman presents with a sudden drop in blood pressure (40/20 mm Hg), history of long-term glucocorticoid use, and cessation of glucocorticoids 4 days ago. These factors are classic for acute adrenal insufficiency (Addisonian crisis), which occurs when there is an insufficient production of corticosteroids (cortisol) due to adrenal gland dysfunction, often triggered by sudden withdrawal of glucocorticoids after prolonged use.
Key diagnostic clues in this case:
- History of long-term glucocorticoid use for bronchial asthma
- Sudden withdrawal of glucocorticoids (she stopped taking them 4 days ago)
- Severe hypotension (40/20 mm Hg) despite the absence of external fluid loss (no signs of dehydration or hypovolemia)
- Muffled heart sounds, poor pulse volume, which suggest shock (due to adrenal crisis)
- Electrolyte abnormalities:
- Low sodium (117 mmol/L)
- High potassium (6.0 mmol/L)
- Low glucose (3.0 mmol/L)
These electrolyte disturbances (hyponatremia, hyperkalemia, hypoglycemia) are characteristic of adrenal insufficiency.
In acute adrenal insufficiency, there is a lack of cortisol (a crucial stress hormone), leading to hypotension, electrolyte disturbances, and shock. The condition can be triggered by stress (like physical overexertion) and the abrupt cessation of glucocorticoid therapy.
Why Other Options Are Incorrect:
- A. Status asthmaticus – Incorrect.
- Status asthmaticus causes severe respiratory distress, but it would not explain the severe hypotension and the electrolyte imbalances (low sodium, high potassium).
- The absence of respiratory distress and normal skin moisture further suggest that this is not an asthma exacerbation.
- B. Hypovolemic shock – Incorrect.
- Hypovolemic shock typically occurs due to blood loss or fluid loss (e.g., from vomiting, diarrhea, or bleeding), but there are no signs of dehydration or significant fluid loss in this case.
- The skin is normal in color and moisture, making hypovolemic shock less likely.
- C. Cardiogenic shock – Incorrect.
- Cardiogenic shock is usually due to severe heart failure or a myocardial infarction, leading to poor perfusion and hypotension.
- The muffled heart sounds are likely due to shock itself and not due to heart failure in this case.
- Moreover, the electrolyte changes point toward adrenal insufficiency, not cardiogenic shock.
- E. Hypoglycemic coma – Incorrect.
- While hypoglycemia is present in this patient (glucose 3.0 mmol/L), it is likely a result of adrenal insufficiency (due to inadequate cortisol) rather than a primary cause of the coma.
- The low glucose is consistent with adrenal insufficiency, but the cause of the shock is the lack of cortisol production, not hypoglycemia alone.
Conclusion:
This patient is experiencing an acute adrenal insufficiency crisis (Addisonian crisis), likely triggered by the withdrawal of glucocorticoids after long-term use. The treatment should focus on immediate glucocorticoid replacement (IV hydrocortisone) and fluid resuscitation to correct hypotension and electrolyte disturbances.
57. A 38-year-old woman has bronchial asthma with concomitant type 1 diabetes mellitus. She receives complex therapy that includes salmeterol, beclomethasone, theophylline, cromolyn, and prednisolone. What group of drugs makes it necessary to correct the insulin dose?
A. Chromones
B. Inhaled glucocorticoids
C. Methylxanthines
D. Long-acting B₂-agonists
E. Systemic glucocorticoids
Correct Answer: E. Systemic glucocorticoids
Explanation:
In this case, the patient has bronchial asthma and type 1 diabetes mellitus, and she is receiving a complex therapy regimen, including systemic glucocorticoids (prednisolone). Systemic glucocorticoids have a significant impact on glucose metabolism, potentially causing hyperglycemia, which is why the insulin dose may need to be adjusted.
Effect of systemic glucocorticoids on glucose metabolism:
- Systemic glucocorticoids (like prednisolone) increase insulin resistance, leading to elevated blood glucose levels.
- As a result, the insulin dose may need to be increased to compensate for the insulin resistance induced by glucocorticoids.
Why Other Options Are Incorrect:
- A. Chromones (Cromolyn) – Incorrect.
- Chromones (such as cromolyn sodium) are used to prevent bronchospasm in asthma and do not significantly affect insulin sensitivity or glucose metabolism. Therefore, they do not require adjustments in the insulin dose.
- B. Inhaled glucocorticoids – Incorrect.
- Inhaled glucocorticoids (such as beclomethasone) have a local effect on the lungs and generally have minimal systemic absorption. They do not typically cause hyperglycemia or require insulin dose adjustments in most cases.
- C. Methylxanthines (Theophylline) – Incorrect.
- Theophylline is a bronchodilator that has mild effects on glucose metabolism but does not significantly alter insulin requirements. It is not a primary cause of hyperglycemia in this patient.
- D. Long-acting B₂-agonists (Salmeterol) – Incorrect.
- Long-acting B₂-agonists (such as salmeterol) primarily act as bronchodilators and have minimal direct effects on glucose metabolism. They are not typically associated with changes in insulin dose.
Conclusion:
Systemic glucocorticoids like prednisolone are the drugs that are most likely to necessitate a correction in the insulin dose for patients with type 1 diabetes mellitus due to their effects on insulin resistance and glucose metabolism.
58. A 29-year-old woman with the gestation period of 39-40 weeks was hospitalized into a maternity hospital with vaginal bleeding and sharp abdominal pain that started one hour ago. Objectively, her blood pressure is 180/100 mm Hg. The fetal heart rate cannot be auscultated. Vaginal examination detects hemorrhagic discharge with blood clots. The cervix is smoothed out and fully open. The amniotic sac is intact and constantly tense. The baby is in a cephalic presentation, with the large segment directed towards the entrance into the lesser pelvis. The placental tissue cannot be detected. What is the most likely diagnosis in this case?
A. Rupture of a vaginal varicosity
B. Placental abruption
C. Complete placenta previa
D. Cervical rupture
E. Rupture of the body of the uterus
Correct Answer: B. Placental abruption
Explanation:
This 29-year-old woman presents with vaginal bleeding, sharp abdominal pain, and no fetal heart sounds at 39-40 weeks gestation, which is highly suggestive of placental abruption. This is a serious obstetric complication where the placenta prematurely separates from the uterine wall, leading to bleeding and potentially fetal distress or death.
Key diagnostic clues in this case:
- Vaginal bleeding with blood clots – Common in placental abruption, where blood accumulates between the uterine wall and the placenta.
- Sharp abdominal pain – Often described as sudden, severe, and persistent pain due to the detachment of the placenta.
- High blood pressure (180/100 mm Hg) – Hypertension is a known risk factor for placental abruption.
- No fetal heart tones – Suggests fetal distress or death, commonly seen in severe cases of placental abruption where blood flow to the fetus is compromised.
- Cervix fully dilated, tense amniotic sac – This is likely a sign that labor is either imminent or progressing but does not exclude placental abruption.
Placental abruption can cause sudden, severe bleeding, and is typically associated with the absence of fetal heart sounds, especially if the abruption is severe and results in significant placental separation.
Why Other Options Are Incorrect:
- A. Rupture of a vaginal varicosity – Incorrect.
- Vaginal varicosities cause bleeding, but the pain is usually mild, and they do not lead to sharp abdominal pain or fetal distress.
- The presentation of severe abdominal pain and the absence of fetal heart tones make this diagnosis unlikely.
- C. Complete placenta previa – Incorrect.
- Placenta previa occurs when the placenta covers the cervix. While it can cause vaginal bleeding, the bleeding is typically painless rather than associated with severe abdominal pain.
- The absence of fetal heart tones and severe pain are not characteristic of placenta previa.
- D. Cervical rupture – Incorrect.
- Cervical rupture generally occurs during labor and delivery, often associated with a history of forceful instrumentation or rapid labor.
- The absence of fetal heart tones and the location of bleeding do not fit the clinical picture of a cervical rupture.
- E. Rupture of the body of the uterus – Incorrect.
- Uterine rupture can cause severe abdominal pain, vaginal bleeding, and fetal distress. However, in this case, the amniotic sac is intact, and the cervix is fully dilated, which makes uterine rupture less likely.
- Placental abruption is more likely to present with these findings, particularly with the fetal heart tones not being auscultated.
Conclusion:
This patient’s vaginal bleeding, sharp abdominal pain, elevated blood pressure, and absence of fetal heart tones point to placental abruption, which requires urgent intervention, such as delivery or blood transfusion, depending on the severity of the condition.
59. A 20-year-old woman complains of fever of 39°C, headache in the frontal region, pain in the eyes, photosensitivity, muscle pain, and dry cough. According to the patient’s medical hi- story, she became acutely ill the day before. Objectively, her condition is severe, her face is hyperemic, her eyes are shiny, with injected sclerae. Pulse 96/min, rhythmic. Heart sounds are muffled. Scattered dry crackles can be heard in the lungs. The oropharyngeal mucosa is hyperemic, granular, with dilated vessels. There are no meningeal symptoms. Complete blood count: leukocytes 3.109/L, eosinophils 1%, band neutrophils – 6%, segmented neutrophils 51%, lymphocytes -35%, monocytes 7%. What is the most likely diagnosis in this case?
A. Meningococcal infection
B. Pneumonia
C. Influenza
D. Measles
E. Epidemic typhus
Correct Answer: C. Influenza
Explanation:
This 20-year-old woman presents with fever (39°C), headache, eye pain, photosensitivity, muscle pain, and dry cough. These are classic symptoms of influenza. Influenza is a viral infection that commonly presents with acute onset of fever, headache, myalgia, and respiratory symptoms such as dry cough. The findings of hyperemic face, shiny eyes, and injected sclerae point toward a viral etiology, as do the absence of meningeal signs and the specific mucosal findings in the oropharynx.
Key diagnostic clues in this case:
- Acute onset of fever, headache, eye pain, and muscle pain are very typical of influenza.
- Dry cough and photosensitivity are also common complaints in the early stages of influenza.
- The patient’s hyperemic face and shiny eyes (signs of conjunctival injection) are seen in influenza, which often causes conjunctivitis.
- The mildly elevated pulse (96/min), normal heart sounds, and the absence of significant meningeal symptoms help to rule out other conditions like meningococcal infection and epidemic typhus.
Why Other Options Are Incorrect:
- A. Meningococcal infection – Incorrect.
- Meningococcal infection typically presents with fever, severe headache, neck stiffness, and meningeal signs (e.g., photophobia, nuchal rigidity), which are absent in this case. The patient does not have the classic signs of meningitis or rash seen in meningococcal infections.
- B. Pneumonia – Incorrect.
- Pneumonia can cause fever, cough, and rales in the lungs, but this patient’s symptoms (such as headache, eye pain, photosensitivity, and muscle pain) are more consistent with a viral infection like influenza. The presence of dry crackles in the lungs does not point to a bacterial pneumonia.
- D. Measles – Incorrect.
- Measles presents with a characteristic maculopapular rash (which is not present in this case) and typically has a prodromal phase with cough, coryza, and conjunctivitis. While some symptoms overlap with influenza, measles is also characterized by a Koplik’s spot in the mouth, which is not mentioned here.
- E. Epidemic typhus – Incorrect.
- Epidemic typhus is caused by Rickettsia and often presents with fever, headache, and a rash that starts on the trunk and spreads. This patient’s presentation does not fit the typical rash pattern seen in typhus. Also, epidemic typhus usually has a longer prodrome, and this patient’s acute onset of symptoms is more suggestive of a viral infection.
Conclusion:
The combination of fever, headache, muscle pain, dry cough, and hyperemic conjunctiva in a young adult with an acute onset is most consistent with influenza. Further confirmation could be made with rapid influenza testing or PCR, but the clinical presentation points strongly toward this diagnosis.
60. A 9-month-old child with an acute respiratory viral infection was receiving antipyretics in a syrup for 2 days. On the second day, the parents note that their child has developed skin redness, neck edema, hoarseness of the voice, barking cough, difficulty breathing, and agitation. In the family’s medical history, the atopic anamnesis is complicated on the father’s side. What is the most likely diagnosis in this case?
A. Acute stenosing laryngitis
B. Quincke’s edema
C. Acute epiglottitis
D. Bronchiolitis
E. Urticaria
Correct Answer: A. Acute stenosing laryngitis
Explanation:
The child is presenting with a combination of symptoms, including skin redness, neck edema, hoarseness of voice, barking cough, difficulty breathing, and agitation, which strongly suggest acute stenosing laryngitis (also known as croup). This condition is commonly caused by a viral infection, often following an acute respiratory viral infection (like the one mentioned in the question), and can lead to swelling of the larynx, causing airway obstruction.
Key features supporting acute stenosing laryngitis (croup):
- Barking cough: This is a classic symptom of croup, which typically sounds like a seal-like bark.
- Hoarseness of voice: This results from swelling and inflammation of the larynx.
- Difficulty breathing: Due to the narrowing of the airway, which leads to respiratory distress.
- Agitation: Often a sign of respiratory distress or hypoxia.
- The child’s age (9 months) and the recent upper respiratory viral infection also align with croup, which is common in infants and young children.
Why Other Options Are Incorrect:
- B. Quincke’s edema (Angioedema) – Incorrect.
- Quincke’s edema involves swelling of the deeper layers of the skin (often the face, lips, or extremities), but it is typically painless, does not cause barking cough, and would not lead to the hoarseness or croup-like symptoms seen here. It could cause difficulty breathing if it affects the airway but is not associated with a viral infection or barking cough.
- C. Acute epiglottitis – Incorrect.
- Acute epiglottitis presents with severe, sudden-onset sore throat, difficulty swallowing, fever, and stridor but is less likely to present with a barking cough. It also tends to present more acutely and may result in drooling, which is not mentioned in this case. Croup is more likely given the barking cough and recent respiratory viral infection.
- D. Bronchiolitis – Incorrect.
- Bronchiolitis primarily affects the lower respiratory tract, causing wheezing, respiratory distress, and difficulty breathing. It is typically associated with viral infections like RSV (respiratory syncytial virus) and would not typically present with a barking cough or hoarseness, which are more suggestive of croup.
- E. Urticaria – Incorrect.
- Urticaria (hives) presents with itchy, raised, red welts on the skin. While it can be associated with allergic reactions, it would not cause difficulty breathing, hoarseness, or a barking cough. The child’s symptoms are more consistent with a respiratory issue like croup.
Conclusion:
The most likely diagnosis is acute stenosing laryngitis (croup), given the barking cough, hoarseness, difficulty breathing, and recent acute respiratory viral infection. The child may require nebulized epinephrine, steroids, and supportive care to manage the airway obstruction caused by swelling in the larynx.
61. The parents of a 6-year-old boy complain of pain and paresthesia in their child’s legs, the child quickly tires when walking. Objectively, the following is observed: disproportional development of the muscles of the upper and lower shoulder girdle, hypotonia of the leg muscles, no pulsation on the femoral arteries, significantly weakened pulsation on the abdominal aorta, systolic murmur in the interscapular area. Blood pressure in the arms -140/100 mm Hg, blood pressure in the legs 100/60 mm Hg. X-ray detects severe erosions (usurations) on the ribs. What is the most likely diagnosis in this case?
A. Coarctation of the aorta
B. Kawasaki disease
C. Takayasu disease
D. Femoral endarteritis
E. Aortic aneurysm
Correct Answer: A. Coarctation of the aorta
Explanation:
This 6-year-old boy presents with pain and paresthesia in the legs, muscle hypotonia, and disproportional development of the shoulder girdle relative to the legs. The clinical findings, including no pulsation in the femoral arteries, weakened abdominal aorta pulsation, systolic murmur in the interscapular area, and significant difference in blood pressure between the arms and legs, are highly suggestive of coarctation of the aorta.
Key diagnostic clues supporting coarctation of the aorta:
- Disproportional muscle development (upper vs. lower body): Coarctation of the aorta can cause impaired blood flow to the lower extremities, leading to muscle weakness and hypotonia in the legs.
- Absent or weak femoral pulses and weakened abdominal aorta pulsation: A hallmark sign of aortic coarctation, where the constricted segment of the aorta causes reduced blood flow to the lower body.
- Systolic murmur in the interscapular area: A murmur is often heard in coarctation of the aorta, typically from turbulent blood flow at the site of narrowing.
- Blood pressure discrepancy: Elevated blood pressure in the upper extremities (140/100 mm Hg) and low blood pressure in the legs (100/60 mm Hg) is a classic sign of coarctation of the aorta, which causes hypertension in the upper body and hypotension in the lower body.
- X-ray finding of erosions on the ribs: Rib notching is commonly seen in long-standing coarctation of the aorta due to the development of collateral circulation in response to the narrowing.
Why Other Options Are Incorrect:
- B. Kawasaki disease – Incorrect.
- Kawasaki disease is a systemic vasculitis affecting children and can cause fever, rash, and coronary artery involvement. However, it typically does not cause muscle hypotonia, disproportional development of muscle mass, or the specific findings of aortic narrowing seen here, like the systolic murmur and rib erosions.
- C. Takayasu disease – Incorrect.
- Takayasu arteritis is a chronic inflammatory disease affecting the large arteries (particularly the aorta). It can cause similar findings such as weak pulses and hypertension, but it typically affects older children and adults and would be more likely to present with systemic inflammatory signs, such as fever and weight loss, which are not seen in this case.
- D. Femoral endarteritis – Incorrect.
- Femoral endarteritis refers to inflammation or infection of the femoral arteries, which would typically present with pain, swelling, or redness in the legs, but would not explain the systolic murmur, hypertension in the arms, or X-ray findings of rib erosions.
- E. Aortic aneurysm – Incorrect.
- An aortic aneurysm may present with pain and vascular compromise, but it typically occurs in adults, especially those with risk factors like atherosclerosis. This child’s presentation is more consistent with coarctation rather than an aneurysm, as aortic aneurysms are less likely to cause a systolic murmur and disproportional muscle development as seen here.
Conclusion:
The most likely diagnosis is coarctation of the aorta, a congenital condition characterized by narrowing of the aorta, leading to upper body hypertension, lower body hypotension, and signs like absent or weak pulses in the lower extremities. The rib notching seen on the X-ray is a classic finding of long-standing coarctation. Early diagnosis and treatment are essential to prevent complications like heart failure and aortic rupture.
62. A 57-year-old patient complains of loss of appetite, heaviness and distension in the epigastric region that are not associated wi- th eating, belching with musty-smelling air and food, nausea, and meat intolerance. According to the patient’s medical history, he has been suffering from peptic ulcer disease of the stomach for the last 18 years, the deterioration of his condition occurred two months ago. Blood test reveals moderately severe anemia, leukocytosis, and increased ESR. Fecal occult blood test is positive. Gastric secretion test detects hypochlorhydria. Esophagogastroduodenoscopy allowed di- agnosing an ulcer 2.5 cm in diameter with uneven, indistinct edges in the gastric cardia on the large curvature. What complication has most likely occurred in the patient?
A. Recurrent bleeding from the gastric ulcer
B. Pyloroduodenal stenosis
C. Gastric ulcer penetration into the tissues of the extraperitoneal space
D. Perivisceritis
E. Malignization of the gastric ulcer
Correct Answer: E. Malignization of the gastric ulcer
Explanation:
The patient presents with loss of appetite, heaviness and distension in the epigastric region, belching with musty-smelling air, nausea, and meat intolerance, all of which are indicative of gastric issues. His medical history includes peptic ulcer disease (PUD) for the past 18 years, with a worsening condition in the last two months. Additionally, the blood test shows moderately severe anemia, leukocytosis, and increased ESR, and the fecal occult blood test is positive, suggesting ongoing bleeding or occult blood loss.
The most concerning finding is the presence of an ulcer 2.5 cm in diameter with uneven, indistinct edges in the gastric cardia. These characteristics—especially the uneven and indistinct edges—are concerning for malignization of the ulcer, where a benign ulcer transforms into a gastric cancer. The long-standing history of peptic ulcer disease and the positive occult blood test further support the suspicion of malignancy, as ulcers that do not heal or present with these irregular features are often considered suspicious for gastric carcinoma.
Key clues supporting malignization:
- Long-standing peptic ulcer disease (18 years): Chronic ulcers have a higher risk of becoming malignant.
- Indistinct ulcer edges: A hallmark feature of malignant ulcers.
- Positive fecal occult blood test: Suggestive of chronic bleeding, which is a common sign of malignancy in gastric ulcers.
- Anemia, leukocytosis, and increased ESR: These findings can be indicative of chronic blood loss and inflammation, which are consistent with gastric cancer.
Why Other Options Are Incorrect:
- A. Recurrent bleeding from the gastric ulcer – Incorrect.
- Recurrent bleeding from a gastric ulcer typically presents with hematemesis or melena and would not necessarily cause anemia, leukocytosis, and positive occult blood tests in the absence of active bleeding. The ulcer described is more likely to be malignant due to the uneven, indistinct edges.
- B. Pyloroduodenal stenosis – Incorrect.
- Pyloroduodenal stenosis usually presents with symptoms of nausea, vomiting, and distension due to impaired gastric emptying. However, the patient’s symptoms—such as meat intolerance and the ulcer findings—are more consistent with a gastric malignancy rather than a stenosis, which would show up with a different clinical presentation.
- C. Gastric ulcer penetration into the tissues of the extraperitoneal space – Incorrect.
- Penetration of a gastric ulcer usually causes severe acute pain, often radiating to the back. While this is a serious complication, it would not explain the chronic symptoms, the positive fecal occult blood test, or the uneven edges of the ulcer, which are more characteristic of gastric cancer.
- D. Perivisceritis – Incorrect.
- Perivisceritis refers to the inflammation of the tissue around the organs (usually due to peritonitis or a bowel perforation). While the patient has some gastrointestinal symptoms, this diagnosis is less likely in the absence of acute, severe abdominal pain and signs of peritonitis, and it does not explain the chronic symptoms or the gastric ulcer findings.
Conclusion:
The most likely diagnosis is malignization of the gastric ulcer, which is suggested by the long-standing peptic ulcer disease, uneven and indistinct ulcer edges, and positive occult blood test. Further diagnostic evaluation, including biopsy and imaging, is crucial for confirming gastric carcinoma and initiating appropriate treatment.
63. A 36-year-old patient was hospitalized on the seventh day after the onset of the di- sease with complaints of marked weakness, fever, headache, and pain in the calf muscles. Objectively, the skin and sclera are icteric, a hemorrhagic rash is observed on the skin. Di- uresis 200 mL. According to the patient’s history, the patient went fishing 2 weeks ago. What is the most likely diagnosis in this case?
A. Brucellosis
B. Trichinellosis
C Leptospirosis
D. Salmonellosis
E. Yersiniosis
Correct Answer: C. Leptospirosis
Explanation:
The patient presents with a combination of symptoms that are characteristic of leptospirosis, including:
- Marked weakness, fever, headache, and pain in the calf muscles, which are common symptoms of leptospirosis.
- Icteric skin and sclera: This suggests jaundice, which is a common manifestation in the icteric phase of leptospirosis.
- Hemorrhagic rash: A skin rash with hemorrhagic features is also seen in severe cases of leptospirosis.
- Low diuresis (200 mL): This could indicate kidney involvement (acute kidney injury), which is a common complication of leptospirosis.
- The history of the patient going fishing two weeks ago suggests a possible exposure to contaminated water or animal urine, which is a known risk factor for leptospirosis, as the disease is often transmitted via contact with water contaminated by animal urine.
Key diagnostic clues supporting leptospirosis:
- Fever, headache, and muscle pain (particularly in the calf muscles) are classic symptoms of leptospirosis.
- Icterus (yellowing of the skin and sclera) and hemorrhagic rash are associated with severe cases of leptospirosis.
- Exposure to contaminated water (fishing, swimming, etc.) is a key risk factor for leptospirosis.
Why Other Options Are Incorrect:
- A. Brucellosis – Incorrect.
- While brucellosis can present with fever, weakness, and joint pain, it typically does not cause hemorrhagic rashes or jaundice as prominent features. Additionally, brucellosis is more commonly acquired through contact with infected animals or their products, rather than contaminated water.
- B. Trichinellosis – Incorrect.
- Trichinellosis usually presents with symptoms like muscle pain, fever, and edema, but jaundice and hemorrhagic rashes are not typical features. The exposure history in this case (fishing) does not support trichinellosis, which is typically associated with consumption of undercooked pork or wild game.
- D. Salmonellosis – Incorrect.
- Salmonellosis typically causes gastrointestinal symptoms (e.g., diarrhea, abdominal pain), which are absent in this patient. While it can cause fever and weakness, it does not typically cause jaundice or hemorrhagic rash, which makes leptospirosis a more likely diagnosis.
- E. Yersiniosis – Incorrect.
- Yersiniosis typically presents with gastrointestinal symptoms (e.g., abdominal pain, diarrhea) and sometimes fever and lymphadenopathy. Jaundice and hemorrhagic rash are not typical features, and there is no indication of the specific exposure seen in leptospirosis.
Conclusion:
The most likely diagnosis is leptospirosis, which fits with the patient’s symptoms, including fever, headache, muscle pain, jaundice, hemorrhagic rash, and a history of exposure to potentially contaminated water while fishing. Prompt diagnosis and treatment with antibiotics (such as doxycycline) are essential to prevent complications, including liver failure, kidney failure, and severe bleeding.
64. A 28-year-old patient complains of a pustular rash on the skin of both his shoulders, fever of 38.5°C, and feeling unwell. According to the patient’s medical history, the disease onset was three days ago, after he received skin abrasions from wearing a knapsack. Object vely, the skin in the area of both his forearms is hyperemic and edematous. There are multiple small foci of superficial pustular papular rash clèments. Bacteriological testing of the contents of the papules detects Staphylococcus aureus. What is the most likely diagnosis in this case?
A. Furunculosis
B. Pyoderma
C. Erysipelas
D. Epidermolysis
E. Streptoderma
Correct Answer: B. Pyoderma
Explanation:
The patient presents with:
- Pustular rash on the skin of both shoulders, accompanied by fever and general malaise.
- The history of abrasions from wearing a knapsack suggests that skin trauma could have led to an infection.
- The skin in the affected area is hyperemic (red) and edematous (swollen), which are signs of inflammation.
- Multiple small pustules and papules are present, indicating a superficial bacterial infection of the skin.
- Bacteriological testing identifies Staphylococcus aureus, which is a common pathogen in pyoderma, particularly in cases of impetigo or folliculitis.
Pyoderma refers to bacterial skin infections that result in pus formation. The presence of superficial pustules, along with the history of skin trauma and Staphylococcus aureus as the causative organism, supports a diagnosis of pyoderma.
Why Other Options Are Incorrect:
- A. Furunculosis – Incorrect.
- Furunculosis (boils) refers to a deeper skin infection that involves the hair follicles and surrounding tissue, typically presenting as a large, painful, red, and fluctuant nodule. Although Staphylococcus aureus can cause furuncles, the rash described here is more superficial (pustules and papules) rather than the deeper abscesses seen in furunculosis.
- C. Erysipelas – Incorrect.
- Erysipelas is a more acute and deep skin infection, usually caused by Group A Streptococcus. It presents with well-defined, raised borders, a red, shiny appearance of the skin, and is often accompanied by systemic signs of infection (fever, chills). The pustules and Staphylococcus aureus in this case are more consistent with pyoderma than with erysipelas.
- D. Epidermolysis – Incorrect.
- Epidermolysis refers to a condition where the skin is fragile and easily blistered, but it is not typically associated with pustular papular rashes or Staphylococcus aureus infections. This diagnosis does not match the description of the rash and the clinical course.
- E. Streptoderma – Incorrect.
- Streptoderma refers to a skin infection caused by Group A Streptococcus, often presenting as impetigo. However, this patient’s culture results show Staphylococcus aureus, not Streptococcus. Therefore, streptoderma is less likely.
Conclusion:
The most likely diagnosis is pyoderma, a superficial bacterial skin infection typically caused by Staphylococcus aureus. It presents with pustular lesions, often following skin trauma, which fits this patient’s history and clinical findings..
65. A 30-year-old woman complains of no menstruations occurring over the last year. She has history of a massive postpartum hemorrhage. Objectively, her secondary sex organs are normally developed, the hair growth pattern is of the female type. Bimanual examination detects normal uterus and uteri- ne appendages. What is a possible cause of amenorrhea in this case?
A. Adrenocortical necrosis
B. Pituitary necrosis
C. Thyroid disorder
D. Ovarian insufficiency
E. Hypothalamic insufficiency
Correct Answer: B. Pituitary necrosis
Explanation:
The patient presents with secondary amenorrhea (absence of menstruation for over a year) following a massive postpartum hemorrhage. The key to understanding this case lies in the fact that a massive postpartum hemorrhage can lead to a condition called Sheehan’s syndrome, which involves pituitary necrosis (damage to the pituitary gland) due to blood loss during childbirth.
In Sheehan’s syndrome, the loss of blood supply to the pituitary gland leads to hypopituitarism, which affects the secretion of several hormones, including those regulating the menstrual cycle (e.g., gonadotropins like LH and FSH). This results in amenorrhea, but with normal development of secondary sexual characteristics, as seen in this patient.
Why Other Options Are Incorrect:
- A. Adrenocortical necrosis – Incorrect.
- Adrenocortical necrosis can lead to adrenal insufficiency and symptoms such as fatigue, hypotension, and electrolyte imbalances, but it does not directly cause amenorrhea or a pattern consistent with the patient’s history. While adrenal insufficiency can impact reproductive health, it is less likely to cause isolated secondary amenorrhea without other adrenal-related symptoms.
- C. Thyroid disorder – Incorrect.
- While thyroid disorders (such as hypothyroidism) can cause secondary amenorrhea, this patient’s secondary sexual characteristics are normal, which makes a thyroid disorder less likely. Additionally, thyroid dysfunction typically affects other areas, such as metabolism and weight, and would likely present with other symptoms, which this patient does not exhibit.
- D. Ovarian insufficiency – Incorrect.
- Ovarian insufficiency or premature ovarian failure can cause amenorrhea; however, the patient has normal development of secondary sexual characteristics, and her history suggests the possibility of a pituitary problem rather than a primary ovarian issue. Ovarian insufficiency usually presents with symptoms like hot flashes, vaginal dryness, and low estrogen levels, which are not mentioned here.
- E. Hypothalamic insufficiency – Incorrect.
- Hypothalamic insufficiency can lead to amenorrhea, but it is typically associated with other signs, such as weight loss, stress, or excessive exercise, which are not mentioned in this case. The patient’s condition following postpartum hemorrhage points more toward a pituitary problem than a hypothalamic one.
Conclusion:
The most likely cause of this patient’s secondary amenorrhea is pituitary necrosis, possibly due to Sheehan’s syndrome following a massive postpartum hemorrhage. This leads to hypopituitarism and amenorrhea, while the development of secondary sexual characteristics remains normal.
66. A 55-year-old woman complains of di- arrhea, irritability, and peeling and pigmentati- on of the open areas of her body (neck, hands, and feet). Her history indicates that corn is one of the main components in her dict. The woman rarely eats other vegetables and legumes and does not eat meat or fish. What is the most likely diagnosis in this case?
A. Psoriasis
B. Pellagra
C. Scurvy
D. Beriberi
E. Swift’s dermatitis (biotin-responsive dermatosis)
Correct Answer: B. Pellagra
Explanation:
The patient presents with:
- Diarrhea, irritability, and peeling and pigmentation changes of the open areas of her body (neck, hands, and feet).
- Her dietary history suggests a heavy reliance on corn, with limited intake of other vegetables, legumes, meat, and fish.
These signs are classic for pellagra, a condition caused by niacin (vitamin B3) deficiency. Pellagra is characterized by the 3 D’s:
- Dermatitis: Skin changes like peeling and hyperpigmentation, often in sun-exposed areas (neck, hands, and feet).
- Diarrhea: Gastrointestinal distress, including diarrhea.
- Dementia: Neurological symptoms such as irritability or confusion (not fully present here but can occur).
Corn is deficient in niacin, and without sufficient niacin in the diet, especially if a person is not consuming animal products (like meat or fish), pellagra can develop.
Why Other Options Are Incorrect:
- A. Psoriasis – Incorrect.
- Psoriasis is a chronic inflammatory skin condition that leads to thickened, scaly patches (especially on the elbows, knees, and scalp), not the peeling and pigmentation changes described here. The condition also wouldn’t be associated with diarrhea or irritability.
- C. Scurvy – Incorrect.
- Scurvy results from vitamin C deficiency and typically presents with gums bleeding, loose teeth, bruising, and joint pain, which are not described in this case. The dietary pattern in this patient suggests a niacin deficiency rather than a vitamin C deficiency.
- D. Beriberi – Incorrect.
- Beriberi is caused by a thiamine (vitamin B1) deficiency, which can lead to symptoms such as neuropathy, edema, and cardiac issues. It is unlikely to present with peeling skin, irritability, or diarrhea as the main symptoms. The patient’s history also points to a niacin deficiency rather than thiamine deficiency.
- E. Swift’s dermatitis (biotin-responsive dermatosis) – Incorrect.
- Swift’s dermatitis or biotin-responsive dermatosis occurs due to a biotin (vitamin B7) deficiency. It typically causes red, scaly lesions around the eyes, nose, and mouth, which doesn’t match the peeling and pigmentation described here. The patient’s dietary habits do not strongly suggest biotin deficiency, and pellagra is a more likely diagnosis.
Conclusion:
This patient most likely has pellagra, which results from niacin deficiency. The classic symptoms of diarrhea, dermatitis, and irritability fit the clinical presentation, especially considering her corn-based diet lacking in niacin-rich foods.
67. A 42-year-old patient complains of acute spastic abdominal pain, nausea, vomiting with intestinal contents, abdominal distension that decreases after vomiting, and gas retention. According to the patient’s history, the disease onset was 4 hours ago. Objectively, the following is observed: pulse 110/min, the tongue is dry and coated, the abdomen is asymmetrically distended enlarged upper half, soft to palpation, painful. Auscultation detects acti- ve peristaltic sounds with metallic tinkling, splashing, and gurgling What is the most likely diagnosis in this case?
A. Intestinal obstruction
B. Perforated stomach ulcer
C. Acute cholecystitis
D. Acute pancreatitis
E. Nonspecific ulcerative colitis
Correct Answer: A. Intestinal obstruction
Explanation:
The patient presents with acute spastic abdominal pain, nausea, vomiting with intestinal contents, abdominal distension, and gas retention. The disease onset was rapid, and the symptoms align with a diagnosis of intestinal obstruction. Key points supporting this diagnosis include:
- Vomiting with intestinal contents: This is a common feature of intestinal obstruction. The vomiting typically becomes more severe and may contain fecal material in later stages.
- Abdominal distension: This is a hallmark of obstruction, as the bowel becomes distended with gas and fluids above the site of the blockage.
- Active peristaltic sounds with metallic tinkling, splashing, and gurgling: These sounds suggest a high-level obstruction, where the bowel is trying to move contents through but is unable to due to the blockage. The metallic tinkling sound indicates the presence of air-fluid levels, often seen in bowel obstruction.
- Pain relief after vomiting: In many cases of intestinal obstruction, vomiting may temporarily relieve abdominal distension and discomfort.
Why Other Options Are Incorrect:
- B. Perforated stomach ulcer – Incorrect.
- Perforated ulcer typically presents with acute severe pain (often referred to as a “sudden, sharp” pain), followed by signs of peritonitis (rigid abdomen, rebound tenderness). The history here (vomiting with intestinal contents, abdominal distension, and peristaltic sounds) is more consistent with obstruction than with a perforated ulcer.
- C. Acute cholecystitis – Incorrect.
- Acute cholecystitis typically presents with right upper quadrant pain, fever, nausea, and vomiting. It does not generally cause symptoms like vomiting with intestinal contents or abdominal distension as seen in this case. Moreover, the absence of right upper quadrant tenderness makes cholecystitis less likely.
- D. Acute pancreatitis – Incorrect.
- Acute pancreatitis generally presents with severe epigastric pain radiating to the back, nausea, and vomiting, but it is usually not associated with the intestinal content vomiting, abdominal distension, or metallic tinkling sounds seen in this case. Pancreatitis can cause abdominal tenderness, but it does not typically lead to the pattern of bowel sounds seen in this patient.
- E. Nonspecific ulcerative colitis – Incorrect.
- Ulcerative colitis usually causes bloody diarrhea, abdominal cramping, and may result in chronic symptoms. The presentation here is acute, with vomiting, intestinal contents, and abdominal distension, which are more indicative of a mechanical obstruction rather than an inflammatory bowel condition.
Conclusion:
The most likely diagnosis is intestinal obstruction, as evidenced by the patient’s acute symptoms of vomiting with intestinal contents, abdominal distension, and active peristaltic sounds with metallic tinkling.
68. After giving birth, a 25-year-old woman developed increased weakness in her legs and unsteady walking. She has been suffering from this condition for 6 years already. Every autumn, she notes a deterioration of her condition. Objectively, the woman is euphoric and exhibits a reduced critical attitude towards her condition. She has horizontal nystagmus, high tendon reflexes, foot clonus, pathological foot reflexes, no abdominal reflexes, ataxia during the Romberg test, and intention tremor and missing during coordination tests. Temporal pallor of the optic discs is observed on the eye fundus. What is the most likely diagnosis in this case?
A. Dyscirculatory encephalopathy
B. Multiple sclerosis
C. Amyotrophic lateral sclerosis
D. Acute disseminated encephalomyelitis
E. Myasthenia gravis
Correct Answer: B. Multiple sclerosis
Explanation:
The patient’s symptoms—increased weakness in the legs, unsteady walking, horizontal nystagmus, high tendon reflexes, foot clonus, pathological foot reflexes, ataxia, intention tremor, and temporal pallor of the optic discs—are highly suggestive of multiple sclerosis (MS). The following points support this diagnosis:
- Chronic progressive weakness and ataxia: MS commonly presents with progressive neurological symptoms, such as weakness and coordination difficulties. The patient’s worsening condition every autumn is typical of the relapsing-remitting nature of MS, where symptoms worsen periodically and then improve.
- Horizontal nystagmus: This is a classic finding in MS, especially in the context of demyelination affecting the pathways involved in eye movement control.
- High tendon reflexes, foot clonus, pathological reflexes: These findings point to upper motor neuron involvement, a common feature of MS due to demyelination of the corticospinal tract.
- Ataxia during the Romberg test and intention tremor: These are signs of cerebellar involvement, which is also typical in MS due to demyelination in the cerebellum and its pathways.
- Temporal pallor of the optic discs: This suggests optic neuritis, which is another common manifestation of MS. Inflammation and demyelination of the optic nerve can lead to visual disturbances and optic disc pallor.
Why Other Options Are Incorrect:
- A. Dyscirculatory encephalopathy – Incorrect.
- Dyscirculatory encephalopathy (chronic cerebral circulatory disorders) typically presents with progressive cognitive decline, dizziness, and memory impairments over time, but it does not cause the specific neurological findings like nystagmus, intention tremor, and optic disc pallor seen in MS.
- C. Amyotrophic lateral sclerosis (ALS) – Incorrect.
- ALS presents with progressive muscle weakness, atrophy, and bulbar symptoms (difficulty speaking or swallowing), but it does not cause the nystagmus, cerebellar signs, or optic nerve changes seen in this patient. Additionally, ALS typically does not have a relapsing-remitting course.
- D. Acute disseminated encephalomyelitis (ADEM) – Incorrect.
- ADEM usually follows a viral infection or vaccination, causing acute demyelination that leads to encephalopathy and neurological deficits. It does not typically present with a chronic course and recurrent worsening as seen in this patient. The presence of optic disc pallor and long-term symptoms suggests MS rather than ADEM.
- E. Myasthenia gravis – Incorrect.
- Myasthenia gravis presents with muscle weakness that worsens with activity and improves with rest, especially affecting the ocular muscles and bulbar muscles. It does not cause the neurological signs (like nystagmus, intention tremor, or optic disc pallor) seen in this case.
Conclusion:
The most likely diagnosis is multiple sclerosis, given the progressive neurological symptoms, episodic worsening of the condition, upper motor neuron signs, cerebellar involvement, and optic nerve changes (optic disc pallor).
69. A 64-year-old man is undergoing outpatient treatment for ischemic heart disease, diffuse cardiosclerosis, persistent atrial fibrillation, and functional class III heart failure. The pharmacotherapy consists of amiodarone twice a day, torasemide every other day, and trimetazidine twice a day. The family physician recommended taking warfarin (3 mg per day) to prevent thrombus formation. What parameter should be measured in this case for the optimal control of the effectiveness and safety of the anticoagulant therapy?
A. Lee-White clotting time
B. Platelet count
C. Erythrocyte sedimentation rate
D. International normalized ratio
E. Fibrinogen levels
Correct Answer: D. International normalized ratio (INR)
Explanation:
The patient is prescribed warfarin to prevent thrombus formation, which is commonly used in conditions like atrial fibrillation to reduce the risk of stroke due to thromboembolism. The effectiveness and safety of warfarin therapy must be monitored regularly because it can have a narrow therapeutic window, meaning that too little warfarin may not be effective, while too much can increase the risk of bleeding.
The International Normalized Ratio (INR) is the most accurate and widely used method to monitor warfarin therapy. The INR standardizes the measurement of prothrombin time (PT) across different laboratories, accounting for variations in the testing process. It provides a consistent way to adjust warfarin doses to maintain an appropriate level of anticoagulation. The target INR range depends on the patient’s condition (usually 2.0 to 3.0 for atrial fibrillation or prevention of stroke).
Why Other Options Are Incorrect:
- A. Lee-White clotting time – Incorrect.
- The Lee-White clotting time is a laboratory test used to measure whole blood clotting time, but it is not typically used to monitor warfarin therapy. It is less specific and standardized compared to INR, which is the preferred method for oral anticoagulation monitoring.
- B. Platelet count – Incorrect.
- Platelet count is important for assessing platelet function or thrombocytopenia, but it is not useful for monitoring the anticoagulation effect of warfarin. Warfarin primarily affects the coagulation cascade, not platelet function.
- C. Erythrocyte sedimentation rate (ESR) – Incorrect.
- ESR is a nonspecific marker of inflammation and is not relevant to monitoring the effectiveness or safety of warfarin therapy. It would not provide any information about anticoagulation levels or bleeding risk.
- E. Fibrinogen levels – Incorrect.
- Fibrinogen levels are useful for assessing coagulation status and inflammation, but they are not typically used to monitor warfarin therapy. INR is the specific test for monitoring anticoagulation with warfarin.
Conclusion:
The most appropriate parameter to monitor the effectiveness and safety of warfarin therapy is the International Normalized Ratio (INR), as it directly measures the anticoagulant effect and helps guide dosage adjustments.
70. A 26-year-old patient complains of inability to open his mouth, fever of 40°C, and sharp pain in the throat on the right. The pain radiates to the left ear. Objectively, noticeable trismus of the masticatory muscles and asymmetry of the right side are observed. The left palatine tonsil is hyperemic and di- splaced towards the middle of the oropharynx, the uvula is displaced to the left. Infiltration, bad breath, and hypersalivation are observed. Retromandibular lymph nodes on the right are enlarged and painful during palpation. Rhinoscopic and otoscopic presentations are normal. What is the most likely diagnosis in this case?
A. Right-sided sialadenitis
B. Right-sided parapharyngeal abscess
C. Submandibular lymphadenitis
D.Right-sided peritonsillar abscess
E. Tumor of the right palatine tonsil
Correct Answer: B. Right-sided parapharyngeal abscess
Explanation:
The patient presents with several key symptoms that point to a parapharyngeal abscess, including:
- Inability to open the mouth (trismus) – This is a classic sign of infection affecting the masticatory muscles, often seen in cases of deep throat infections like a parapharyngeal abscess.
- Fever of 40°C, sharp pain in the throat on the right – These symptoms suggest severe infection.
- Pain radiating to the left ear – This is common with infections in the pharyngeal and peritonsillar regions.
- Asymmetry of the right side, displaced uvula – Displacement of the uvula towards the unaffected side (left) is a classic sign of peritonsillar abscess but can also occur in parapharyngeal abscess due to nearby tissue swelling.
- Infiltration, bad breath (halitosis), and hypersalivation – Indicative of acute infection in the throat.
- Enlarged and painful retromandibular lymph nodes – Lymphadenopathy is common in throat infections, especially in deeper neck spaces like the parapharyngeal space.
Why Other Options Are Incorrect:
- A. Right-sided sialadenitis – Incorrect.
- Sialadenitis typically presents with painful swelling of the salivary glands, usually accompanied by dry mouth and pus discharge. It is unlikely to cause the trismus and tonsillar displacement seen here.
- C. Submandibular lymphadenitis – Incorrect.
- Lymphadenitis in the submandibular region might cause pain and swelling of lymph nodes, but it would not explain the trismus, tonsillar displacement, or the overall severe infection affecting the throat and mouth as seen in this patient.
- D. Right-sided peritonsillar abscess – Incorrect.
- A peritonsillar abscess would present with similar symptoms, including tonsillar displacement and trismus, but typically the tonsil would be more visibly affected, with swelling and pus. Additionally, uvula displacement is more commonly seen with parapharyngeal abscess than with peritonsillar abscess.
- E. Tumor of the right palatine tonsil – Incorrect.
- A tumor would not typically present with fever, trismus, and the acute inflammatory signs seen here. A parapharyngeal abscess is a more likely cause for the severe, acute symptoms.
Conclusion:
The most likely diagnosis in this case is a right-sided parapharyngeal abscess, which explains the fever, pain, trismus, tonsillar displacement, and lymphadenopathy, all suggestive of an acute deep neck space infection.
71. A 25-year-old woman complains of discomfort in the area of her external genitalia. The discomfort developed a few days ago. Objectively, in the area of the left labia, there is a painless red ulcerative skin lesion with clear and smooth edges, dense at its base. Painless enlargement of the right inguinal regional lymph node was detected. What is the most likely diagnosis in this case?
A Primary syphilis
B. Carbuncle
C. Genital herpes
D. Bartholinitis
E. Candidiasis
Correct Answer: A. Primary syphilis
Explanation:
The patient presents with a painless ulcer on the external genitalia with clear and smooth edges and a dense base, along with painless inguinal lymphadenopathy. These are classic signs of a syphilitic chancre, which occurs in primary syphilis (caused by Treponema pallidum).
Key Features of Primary Syphilis:
- Painless, well-demarcated ulcer (chancre)
- Smooth, indurated (firm) base
- Painless, unilateral inguinal lymphadenopathy
- Heals spontaneously within 3–6 weeks without treatment
Why Other Options Are Incorrect:
- B. Carbuncle – Incorrect.
- A carbuncle is a painful, pus-filled infection involving multiple hair follicles, typically caused by Staphylococcus aureus. It presents as a deep, red, swollen, and tender area with purulent drainage, not as a painless ulcer.
- C. Genital herpes – Incorrect.
- Herpes simplex virus (HSV) causes painful, multiple vesicles that rupture to form shallow ulcers with erythematous bases. These are usually painful and recurrent, unlike the painless chancre of syphilis.
- D. Bartholinitis – Incorrect.
- Bartholin gland infection causes unilateral, painful swelling near the vaginal opening due to an abscess. It does not present as a painless ulcer with inguinal lymphadenopathy.
- E. Candidiasis – Incorrect.
- Vulvovaginal candidiasis causes pruritus (itching), thick white discharge, and erythema. It does not cause an ulcer or painless lymphadenopathy.
Conclusion:
The most likely diagnosis is primary syphilis, characterized by a painless genital ulcer (chancre) with indurated edges and painless regional lymphadenopathy.
72. A 16-year-old girl complains of heavy bleeding observed for 4 days after a two month menstruation delay, irregular menstrual cycle, and opsomenorrhea. According to the patient’s medical history, her menarche occurred at the age of 15. What is the most likely diagnosis in this case?
A. Cervical polyp
B. Subepithelial cervical endometriosis
C. Juvenile uterine bleeding
D. Cancer of the body of the uterus
E. Acute leukemia
Correct Answer: C. Juvenile uterine bleeding
Explanation:
The patient is a 16-year-old girl with:
- Heavy bleeding for 4 days after a 2-month delay in menstruation
- Irregular menstrual cycles
- Opsomenorrhea (infrequent menstruation)
- Menarche at 15 years old
These features suggest anovulatory cycles, which are common in adolescents due to an immature hypothalamic-pituitary-ovarian (HPO) axis. Juvenile uterine bleeding (JUB) refers to abnormal uterine bleeding in adolescents caused by hormonal dysregulation, often after menarche.
Why Other Options Are Incorrect:
- A. Cervical polyp – Incorrect.
- Cervical polyps cause spotting or postcoital bleeding, not heavy, prolonged menstrual bleeding with cycle irregularities.
- B. Subepithelial cervical endometriosis – Incorrect.
- Endometriosis typically presents with pelvic pain, dysmenorrhea, and dyspareunia rather than irregular heavy bleeding in a newly menstruating adolescent.
- D. Cancer of the body of the uterus – Incorrect.
- Endometrial cancer is extremely rare in adolescents and usually presents in postmenopausal or older women with risk factors like obesity, PCOS, or unopposed estrogen exposure.
- E. Acute leukemia – Incorrect.
- While leukemia can cause abnormal bleeding due to thrombocytopenia, it is typically accompanied by petechiae, pallor, fatigue, and frequent infections, which are not mentioned in this case.
Conclusion:
The most likely diagnosis is juvenile uterine bleeding (JUB), which results from anovulatory cycles due to an immature HPO axis, commonly seen in adolescents..
73. A 35-year-old woman, gravida 1, is at 39 weeks of her full-term pregnancy. The duration of the second stage of labor is 1.5 hours. Objectively, the dimensions of the pelvis are 26-29-32-20, the expected weight of the fetus -4400 g, contractions are active, the fetus is in a cephalic presentation with the head pressed to the entrance to the lesser pelvis, Henkel- Vasten sign is positive. The fetal heart rate is 118-126/min, sonorous, rhythmic. Vaginal examination detects complete opening of the cervix, the head is one finger-width in the lesser pelvis, severe caput succedaneum and edema of the vulva are observed. What complication has occurred during the childbirth?
A. Discoordinated labor activity
B. Weak labor activity
C. Placental abruption
D. Uterine rupture
E Clinically narrow pelvis
Correct Answer: E. Clinically narrow pelvis
Explanation:
The patient is a 35-year-old primigravida at 39 weeks of gestation, experiencing prolonged second stage of labor (1.5 hours) with:
- Cephalic presentation, but fetal head is still at the pelvic inlet
- Henkel-Vasten sign positive (suggests a discrepancy between fetal head size and pelvic dimensions)
- Severe caput succedaneum and vulvar edema (indicates prolonged labor with obstruction)
- Pelvic dimensions: 26-29-32-20 (suggesting possible pelvic contraction)
- Large fetus (~4400 g) (macrosomia increases the risk of labor obstruction)
These findings strongly suggest a clinically narrow pelvis, meaning the fetal head cannot descend properly due to cephalopelvic disproportion (CPD).
Why Other Options Are Incorrect:
- A. Discoordinated labor activity – Incorrect.
- Discoordinated labor presents with irregular, uncoordinated contractions leading to prolonged labor but does not necessarily involve fetal head arrest at the pelvic inlet.
- B. Weak labor activity – Incorrect.
- The case describes active contractions. Weak labor (hypotonic dysfunction) would present with ineffective contractions and prolonged cervical dilation, not a fetal head that remains high.
- C. Placental abruption – Incorrect.
- Abruptio placentae presents with painful vaginal bleeding, uterine tenderness, and fetal distress (noted by late decelerations or bradycardia), which are not mentioned here.
- D. Uterine rupture – Incorrect.
- Uterine rupture presents with sudden pain, loss of contractions, fetal distress, and maternal instability (hypotension, tachycardia). The fetal heart rate (118-126/min) remains stable, which makes rupture unlikely.
Conclusion:
This is a case of clinically narrow pelvis (CPD), where the fetal head is unable to descend due to a mismatch between the pelvis and the fetal size. This is a strong indication for cesarean delivery to prevent fetal and maternal complications.
74. A 5-year-old child became acutely ill wi- th fever of 39.2°C, one episode of vomiting, complaints of cramping pain in the abdomen, tenesmus, and frequent bowel movements that produce a small amount of feces and a large amount of mucus with pus and blood streaks. Examination detects a dense sigmoid colon that is painful to palpation. What is the most likely diagnosis in this case?
A. Acute appendicitis
B. Cholera
C. Shigellosis
D. Rotavirus infection
E. Salmonellosis
Correct Answer: C. Shigellosis
Explanation:
Shigellosis (bacillary dysentery) is a bacterial infection caused by Shigella spp., commonly affecting children. The hallmark symptoms include:
- Acute onset of high fever (39.2°C)
- Abdominal pain and tenesmus (painful straining to defecate)
- Frequent bowel movements with small fecal volume
- Presence of mucus, pus, and blood in stool (classic sign of dysentery)
- Sigmoid colon tenderness and thickening (due to inflammation and spasm)
These features strongly suggest shigellosis, a common cause of infectious colitis in young children.
Why Other Options Are Incorrect:
- A. Acute appendicitis – Incorrect.
- Appendicitis presents with localized right lower quadrant pain (McBurney’s point tenderness), nausea, and anorexia. Diarrhea with mucus and blood is uncommon in appendicitis.
- B. Cholera – Incorrect.
- Cholera causes massive watery diarrhea (“rice-water stools”), severe dehydration, and no tenesmus or significant abdominal pain. The presence of mucus, pus, and blood suggests an invasive bacterial infection (not cholera).
- D. Rotavirus infection – Incorrect.
- Rotavirus mainly affects infants and presents with watery diarrhea, vomiting, and dehydration, but does not cause mucus, pus, or blood in stools.
- E. Salmonellosis – Incorrect.
- Salmonella gastroenteritis presents with profuse diarrhea, fever, and abdominal cramping, but bloody diarrhea with tenesmus is uncommon.
Conclusion:
This is a classic case of shigellosis, characterized by fever, bloody diarrhea with mucus and pus, tenesmus, and sigmoid colon involvement. Early antibiotic treatment (e.g., azithromycin or ciprofloxacin) and fluid replacement are necessary to prevent complications.
75 A 38-year-old woman complains of pustular rash on her body, progressive growth of hair on her body and face, and constant headache. Objectively, the following is observed: height 164 cm, body weight -102 kg, uneven distribution of adipose tissue mainly on the stomach, trunk, and face, hypotrophy of the muscles of the buttocks and thighs. Blood pressure 205/110 n mm Hg. Hg pulse 105/min. Blood test: glucose- 11 mmol/L, sodium 155 mmol/L, potassium 3.7 mmol/L, total cholesterol 6.5 mmol/L. Blood cortisol in the morning 720 nmol/L. ACTH-87 pg/mL. What is the most likely diagnosis in this case?
A. Hyperparathyroidism
B. Cushing disease
C. Primary alimentary obesity
D. Pheochromocytoma
E. Diabetes insipidus
Correct Answer: B. Cushing Disease
Explanation:
Cushing disease is a condition caused by an ACTH-secreting pituitary adenoma, leading to excess cortisol production. This patient presents with classic features of hypercortisolism:
- Pustular rash (due to immune suppression and hyperandrogenism)
- Hirsutism (excess hair growth due to androgen excess)
- Central obesity (fat accumulation on the abdomen, trunk, and face)
- “Moon face” and “buffalo hump” (due to altered fat distribution)
- Hypertension (205/110 mmHg) (due to cortisol-induced vascular sensitivity)
- Hyperglycemia (glucose 11 mmol/L) (cortisol induces insulin resistance)
- Muscle wasting (buttocks, thighs) (due to protein catabolism)
- Electrolyte imbalances:
- Hypernatremia (Na⁺ 155 mmol/L)
- Hypokalemia (K⁺ 3.7 mmol/L) (though mild, it is common in Cushing’s)
Laboratory findings:
- Elevated cortisol (720 nmol/L in the morning) → Suggests Cushing syndrome
- High ACTH (87 pg/mL) → Confirms Cushing disease (ACTH-dependent Cushing syndrome)
Why Other Options Are Incorrect:
- A. Hyperparathyroidism – Incorrect.
- Hyperparathyroidism presents with hypercalcemia, kidney stones, bone pain, and abdominal symptoms (“bones, stones, abdominal groans, and psychic moans”), which are absent here.
- C. Primary alimentary obesity – Incorrect.
- While obesity is present, Cushing’s disease causes muscle atrophy, hypertension, hyperglycemia, and hirsutism, which are not seen in simple obesity.
- D. Pheochromocytoma – Incorrect.
- Pheochromocytoma causes paroxysmal hypertension, sweating, palpitations, and headache due to catecholamine excess, but does not cause central obesity or hirsutism.
- E. Diabetes insipidus – Incorrect.
- Diabetes insipidus is characterized by polyuria, polydipsia, and dehydration due to ADH deficiency, but not obesity, hypertension, or hirsutism.
Conclusion:
This is a classic presentation of Cushing disease, with ACTH-dependent hypercortisolism, leading to hypertension, hyperglycemia, truncal obesity, and hirsutism. The next step would be a dexamethasone suppression test and pituitary MRI to confirm the diagnosis.
76. An 18-year-old patient complains of li- quid foul-smelling discharge from her vagina, discomfort during sexual life. Her menstrual cycle is irregular. Examination of the external genitalia detects hyperemia and edema of the vulva. Examination in the mirrors shows that the vaginal mucosa is hyperemic, the cervix is clean, the discharge is profuse and foamy. What is the most likely diagnosis in this case?
A Trichomoniasis
B. Candidiasis
C. Nonspecific vaginitis
D. Chlamydiosis
E. Gonorrhea
Correct Answer: A. Trichomoniasis
Explanation:
Trichomoniasis is a sexually transmitted infection (STI) caused by Trichomonas vaginalis, a flagellated protozoan. The key features in this patient that strongly suggest trichomoniasis include:
- Profuse, foul-smelling, foamy vaginal discharge
- Hyperemia and edema of the vulva
- Hyperemic vaginal mucosa
- Discomfort during intercourse (dyspareunia)
The classic presentation of trichomoniasis is yellow-green, frothy (foamy) discharge with a foul odor, accompanied by vaginal and vulvar irritation.
Why Other Options Are Incorrect:
- B. Candidiasis – Incorrect.
- Candidiasis (yeast infection) typically presents with:
- Thick, white, “cottage cheese-like” discharge (not foamy or foul-smelling)
- Intense itching and burning
- No foul odor
- Candidiasis (yeast infection) typically presents with:
- C. Nonspecific vaginitis (Bacterial vaginosis – BV) – Incorrect.
- BV presents with:
- Thin, grayish-white discharge with a fishy odor
- No foamy discharge
- No significant vulvar or vaginal inflammation
- BV presents with:
- D. Chlamydiosis (Chlamydia infection) – Incorrect.
- Chlamydia is often asymptomatic, but may cause:
- Mild mucopurulent cervical discharge
- Dysuria, lower abdominal pain
- No foamy discharge
- Chlamydia is often asymptomatic, but may cause:
- E. Gonorrhea – Incorrect.
- Gonorrhea typically presents with:
- Purulent yellow-green discharge (but not foamy)
- Cervicitis (red, friable cervix with mucopurulent discharge)
- Dysuria, pelvic pain
- Gonorrhea typically presents with:
Conclusion:
This patient’s profuse, foul-smelling, foamy discharge, vaginal hyperemia, and dyspareunia are highly characteristic of trichomoniasis. The diagnosis can be confirmed by wet mount microscopy, which shows motile trichomonads. Metronidazole is the treatment of choice.
77. A 72-year-old patient has been hospitalized with complaints of intense palpitations. The symptoms started suddenly, approximately 40 minutes ago. The patient denies chest pain or dizziness. He has history of arterial hypertension, for the treatment of which he regularly. takes medicines, but does not know their name. Objectively, the patient is conscious and oriented, verbal contact with him is productive. Blood pressure 104/55 mm Hg, SpO2 – 97% on room air. ECG shows the heart rate of 190/min with a regular rhythm, the width of the QRS complexes is 0.09 seconds, a negative P wave is registered after each ventricular complex. What drug should be used to start the treatment of this patient?
A. Adenosine intravenously
B. Amiodarone intravenously
C Nitroglycerin intravenously
D. Metoprolol per os
E. Verapamil per os
Correct Answer: A. Adenosine intravenously
Explanation:
The patient presents with sudden-onset palpitations, a regular narrow-QRS tachycardia (HR 190/min, QRS width 0.09 sec), and inverted P waves following QRS complexes, which strongly suggests paroxysmal supraventricular tachycardia (PSVT), most likely atrioventricular nodal reentrant tachycardia (AVNRT).
Key ECG findings supporting PSVT:
- Narrow QRS (<120 ms or 0.12 sec) → Suggests supraventricular origin.
- Regular rhythm with HR ~190/min → Typical for AVNRT.
- Inverted P waves after QRS (retrograde P waves) → Due to reentry through the AV node.
- Sudden onset and termination → Characteristic of PSVT.
First-line treatment for PSVT:
Adenosine IV is the drug of choice for acute termination of PSVT. It acts by:
- Transiently blocking AV node conduction
- Interrupting the reentrant circuit
- Restoring normal sinus rhythm
Why Other Options Are Incorrect:
- B. Amiodarone intravenously – Incorrect.
- Amiodarone is used for ventricular tachycardia (VT) or atrial fibrillation with rapid ventricular response, but it is not the first-line drug for PSVT.
- C. Nitroglycerin intravenously – Incorrect.
- Nitroglycerin is not an antiarrhythmic; it is used for angina and heart failure.
- D. Metoprolol per os – Incorrect.
- Beta-blockers (e.g., metoprolol) can prevent recurrence of PSVT but are not first-line for acute termination.
- E. Verapamil per os – Incorrect.
- Verapamil (a calcium channel blocker) is an alternative to adenosine, but it is not first-line and should be used IV, not orally, in emergencies.
Conclusion:
This patient’s acute PSVT (likely AVNRT) should be treated with IV adenosine, which rapidly restores sinus rhythm by transiently blocking the AV node. If ineffective, IV verapamil or beta-blockers can be considered.
78. A 22-year-old pregnant woman has been hospitalized in a severe condition. Throughout the past three days, she developed edemas, headache, nausea, and one episode of vomiting. Objectively, her consciousness is clouded, her blood pressure is 160/130 mm Hg. She presents with small fibrillar twitehing of the facial muscles and problems with nasal breathing. During transportation, the woman’s arms started twitching, her body stretched out, her spine curved, her jaws tightly clenched, and she stopped breathing. Then she developed clonic seizures and marked cyanosis. After that, the seizures stopped, a deep noisy inhale occurred, and blood-stained foam appeared on the patient’s lips. What is the most likely diagnosis in this case?
A. Hypertensive crisis
B. Diabetic coma
C Eclampsia
D. Epilepsy
E. Chorea
Correct Answer: C. Eclampsia
Explanation:
This pregnant woman presents with severe hypertension (160/130 mmHg), edema, headache, nausea, vomiting, clouded consciousness, and seizures, which are classic signs of eclampsia.
Key Features Suggesting Eclampsia:
- Severe preeclampsia symptoms (hypertension, edema, headache, nausea).
- Seizures (tonic-clonic) → Initial tonic phase (body stretched, jaws clenched, apnea, cyanosis) followed by clonic phase (convulsions, blood-stained froth).
- Postictal state → Deep noisy inhale after seizure.
Pathophysiology:
Eclampsia is a severe complication of preeclampsia characterized by seizures unrelated to epilepsy. It results from cerebral vasospasm, ischemia, and increased intracranial pressure, leading to seizures and neurological symptoms.
Management:
- Immediate magnesium sulfate IV to prevent further seizures.
- Control blood pressure with labetalol or hydralazine IV.
- Expedited delivery (definitive treatment).
Why Other Options Are Incorrect:
- A. Hypertensive crisis – Incorrect.
- Hypertensive crises cause severe hypertension but do not present with seizures or postictal states.
- B. Diabetic coma – Incorrect.
- Diabetic ketoacidosis (DKA) or hyperosmolar coma presents with dehydration, fruity breath (DKA), deep Kussmaul breathing, but not seizures.
- D. Epilepsy – Incorrect.
- The patient has no history of epilepsy, and seizures in eclampsia occur in the setting of hypertension and preeclampsia.
- E. Chorea – Incorrect.
- Chorea presents as involuntary, irregular, jerky movements, not tonic-clonic seizures.
Conclusion:
The presence of hypertension, neurological symptoms, and seizures during pregnancy confirms eclampsia, a life-threatening emergency requiring immediate magnesium sulfate and blood pressure control.
79. A 15-year-old girl complains of the absence of menstruations. Objectively, she has a short. stature, a short neck with vertical skin folds on both sides, deformed auricles, and a low line of hair growth on her neck. Her chest is barrel-shaped, her elbow joints are deformed. There are multiple pigment spots on the skin of her torso and limbs. Her mammary glands are absent and the nipples are underdeveloped. Her intelligence is not impaired. There are nо Barr bodies in the cells of her buccal epitheli- um. What is the most likely diagnosis in this case?
A. Down syndrome
B. Turner syndrome.
C. Patau Patau syndr syndrome
D. Klinefelter syndrome
E. Edwards syndrome
Correct Answer: B. Turner syndrome
Explanation:
This 15-year-old girl presents with primary amenorrhea, short stature, webbed neck, barrel-shaped chest, multiple pigmented nevi, and underdeveloped secondary sexual characteristics (absent mammary glands, underdeveloped nipples). These features are characteristic of Turner syndrome (45, X0).
Key Features of Turner Syndrome:
- Genetic cause: 45, X0 karyotype (monosomy X).
- Short stature (due to SHOX gene deletion affecting bone growth).
- Webbed neck (pterygium colli) and low posterior hairline.
- Shield (barrel-shaped) chest with widely spaced nipples.
- Cubitus valgus (deformed elbow joints).
- Multiple pigmented nevi.
- Ovarian dysgenesis → Primary amenorrhea, absent secondary sexual characteristics (hypogonadism, infertility).
- No Barr bodies in buccal epithelial cells (since they lack a second X chromosome).
- Normal intelligence (unlike Down syndrome).
Why Other Options Are Incorrect:
- A. Down syndrome (Trisomy 21) – Incorrect.
- Down syndrome presents with intellectual disability, flat facial profile, upslanted palpebral fissures, single palmar crease, and cardiac defects (e.g., AV septal defect).
- Turner syndrome patients have normal intelligence.
- C. Patau syndrome (Trisomy 13) – Incorrect.
- Features include microcephaly, cleft lip/palate, polydactyly, and congenital heart defects.
- Life expectancy is very short (most die within the first year).
- D. Klinefelter syndrome (47, XXY) – Incorrect.
- Klinefelter syndrome occurs only in males, presenting with tall stature, gynecomastia, small testes, infertility, and learning difficulties.
- Barr bodies are present in Klinefelter syndrome (extra X chromosome).
- E. Edwards syndrome (Trisomy 18) – Incorrect.
- Features include prominent occiput, clenched hands with overlapping fingers, rocker-bottom feet, congenital heart defects.
- Life expectancy is very short (most die within the first year).
Conclusion:
The classic physical features (short stature, webbed neck, widely spaced nipples, primary amenorrhea, ovarian dysgenesis, no Barr bodies) confirm Turner syndrome (45, X0). Hormone replacement therapy (estrogen) is needed to induce puberty and prevent osteoporosis.
- A sanitary-epidemiological station at a kural region received an emergency message that a patient diagnosed with typhoid fever was admitted to the infectious diseases department of the central regional hospital. What is the main transmission route of this disease?
A Water-borne transmission
B. Droplet transmission
C. Food-borne transmission
D. Arthropod-borne transmission
E. Household transmission
Correct Answer: A. Water-borne transmission
Explanation:
Typhoid fever is caused by Salmonella typhi and is primarily transmitted through the fecal-oral route, with contaminated water being the most common vehicle. The infection typically occurs in areas with poor sanitation and unsafe drinking water.
Main Transmission Routes of Typhoid Fever:
- Water-borne transmission (Most common)
- Drinking contaminated water (with feces containing S. typhi).
- Poor sanitation in rural and developing areas.
- Major cause of epidemics in overcrowded areas with inadequate sewage disposal.
- Food-borne transmission (Less common but possible)
- Eating contaminated food (especially raw vegetables, milk, or seafood washed with infected water).
- Food handlers who are asymptomatic carriers (e.g., Typhoid Mary case).
- Household transmission
- Direct person-to-person transmission is rare but can occur via contaminated hands, utensils, or surfaces.
- Occurs when hygiene practices are poor.
Why Other Options Are Incorrect:
- B. Droplet transmission – Incorrect.
- Typhoid fever is not spread by respiratory droplets.
- Diseases like tuberculosis, influenza, and COVID-19 use this route.
- C. Food-borne transmission – Partially correct but not the primary route.
- Contaminated food can transmit typhoid, but water contamination is the most frequent cause in outbreaks.
- D. Arthropod-borne transmission – Incorrect.
- Typhoid fever is not transmitted by insects.
- Diseases like malaria (mosquitoes), Lyme disease (ticks), and plague (fleas) follow this route.
- E. Household transmission – Incorrect as a primary route.
- Direct person-to-person transmission is not a major route; instead, contaminated food or water is responsible.
Conclusion:
The primary mode of transmission of typhoid fever is water-borne (contaminated drinking water). To prevent typhoid, improving sanitation, water purification, hand hygiene, and vaccination is essential.
81. A 28-year-old woman complains of infertility, observed for the last three years. She has history of gonorrhea. Objectively, the development of her genitals is normal. Her basal temperature, observed for three cycles, is biphasic. What is the most likely cause of the infertility in this case?
A Impaired tubal patency
B. Endocrine factor
C. Anomalous structure of the genitals
D. Endometriosis
E. Immunological infertility
Correct Answer: A. Impaired tubal patency
Explanation:
In this case, the most likely cause of infertility is impaired tubal patency, which can be a consequence of previous gonorrhea infection. Gonorrhea is a sexually transmitted infection (STI) caused by Neisseria gonorrhoeae, which can lead to pelvic inflammatory disease (PID) if left untreated. PID often causes scarring and blockage of the fallopian tubes, leading to infertility. This condition is a common cause of female infertility.
Why Other Options Are Less Likely:
- B. Endocrine factor – Unlikely.
- The woman has a biphasic basal temperature over three cycles, which suggests that ovulation is occurring normally. Thus, there is no strong evidence of an endocrine issue such as hormonal imbalances (e.g., thyroid disorders, polycystic ovary syndrome).
- C. Anomalous structure of the genitals – Unlikely.
- The patient has no complaints or signs of abnormal genital development or congenital malformations. The development of her genitals is reported as normal.
- D. Endometriosis – Less likely.
- While endometriosis can lead to infertility, the history of gonorrhea is more likely to lead to tubal damage. Endometriosis typically causes pelvic pain, painful menstruation, and sometimes adhesions or ovarian cysts, but there’s no indication of these symptoms in this case.
- E. Immunological infertility – Unlikely.
- Immunological infertility occurs when the body’s immune system attacks the sperm or embryos, but this is a rare cause of infertility. The woman’s symptoms and history do not point to an immune issue.
Conclusion:
Given the woman’s history of gonorrhea, which is a common cause of PID and tubal damage, impaired tubal patency is the most likely cause of infertility in this case. Treatment and diagnosis would typically involve laparoscopy to assess tubal function, along with possible interventions such as tubal surgery or in vitro fertilization (IVF).
82 A 45-year-old patient complains of cramping abdominal pain, tenesmus, and frequent liquid stools with blood. The disease onset was 5 years ago. Objectively, the following is observed: body temperature 37.5°C, blood pressure 110/60 mm Hg, pulse 98/min, the abdomen is soft, painful during palpation along the course of the large intestine. Irrigography results: the large intestine is narrowed, the haustra are absent, the contours are uneven and indistinct. Complete blood count: crythrocytes 3.2 1012/L, hemoglobin 110 g/L, color index 0.85, leukocytes 8.1109/L, band neutrophils – 5%, lymphocytes 30%, monocytes 3%, eosinophils 3%, segmented neutrophils 59%, ESR 24 mm/hour. Fecal calprotectin -95 units. What drug must be prescribed first in this case?
A. Pancreatin
B. Loperamide
C. Ciprofloxacin
D. Mesalazine
E. Drotaverine
Correct Answer: D. Mesalazine
Explanation:
The patient is presenting with symptoms of chronic inflammatory bowel disease (IBD), likely ulcerative colitis or Crohn’s disease. These conditions cause cramping abdominal pain, tenesmus, and frequent liquid stools with blood, which aligns with the patient’s symptoms.
- Irrigography reveals features of intestinal inflammation: the narrowing of the large intestine, loss of haustra, and uneven contours are suggestive of inflammatory bowel disease.
- The elevated ESR and leukocytosis indicate an inflammatory process.
- Fecal calprotectin is a marker of intestinal inflammation, and the elevated level (95 units) supports the diagnosis of IBD.
The treatment of IBD usually starts with 5-aminosalicylic acid (such as mesalazine) as it is effective for reducing inflammation in the intestines.
Why Other Options Are Less Likely:
- A. Pancreatin – Unlikely.
- Pancreatin is a digestive enzyme used to treat pancreatic insufficiency. The patient’s symptoms are more suggestive of an inflammatory bowel disease rather than a pancreatic disorder.
- B. Loperamide – Unlikely.
- Loperamide is an anti-diarrheal medication, but it is not recommended for IBD as it can worsen the condition by causing bowel stasis. The primary treatment in this case is anti-inflammatory, not symptomatic.
- C. Ciprofloxacin – Unlikely.
- Ciprofloxacin is an antibiotic, but there is no clear evidence of a bacterial infection causing the patient’s symptoms. The diagnosis points to inflammatory bowel disease, not an infectious cause.
- E. Drotaverine – Unlikely.
- Drotaverine is an antispasmodic used for abdominal pain related to spasms. While it can relieve cramping, it does not treat the underlying inflammation in IBD. The first-line treatment for IBD is mesalazine, not antispasmodics.
Conclusion:
Mesalazine is the most appropriate medication to begin treatment for the patient’s condition, as it is commonly used to manage inflammatory bowel disease by reducing intestinal inflammation. Further management may include other medications like corticosteroids or immunosuppressants depending on the severity and response to initial treatment.
83. A 34-year-old patient complains of headache, edema of the limbs and face, and pink-red urine. He has history of acute tonsillitis that occurred three weeks ago. Objectively, the following is observed: pulse – 98/min, blood pressure 180/110 mm Hg. Urinalysis results: specific gravity 1.021, protein – 2.3 g/L, erythrocytes 35-40 in sight, leukocytes -3-5 in sight, granular casts 3-4 in sight. What is the most likely diagnosis in this case?
A. Chronic glomerulonephritis
B. Acute pyelonephritis
C. Hemorrhagic cystitis
D. Acute glomerulonephritis
E. Urolithiasis
Correct Answer: D. Acute glomerulonephritis
Explanation:
The patient presents with headache, edema of the limbs and face, and pink-red urine, which are common symptoms of acute glomerulonephritis (AGN), especially following an acute tonsillitis episode. The key findings include:
- History of recent infection (acute tonsillitis): This is significant as post-streptococcal glomerulonephritis can occur after an upper respiratory tract infection, particularly caused by group A Streptococcus.
- Urinalysis findings:
- Proteinuria (2.3 g/L) indicates kidney involvement.
- Hematuria (35-40 erythrocytes per high-power field) suggests glomerular damage with red blood cell leakage into the urine.
- The presence of granular casts indicates renal tubular damage and is characteristic of glomerulonephritis.
- Elevated blood pressure (180/110 mm Hg) is consistent with acute glomerulonephritis, which can lead to hypertension as a result of renal dysfunction.
Why Other Options Are Less Likely:
- A. Chronic glomerulonephritis – Unlikely.
- Chronic glomerulonephritis usually presents with long-term symptoms such as gradual onset of edema, proteinuria, and hematuria, but typically not after a recent infection. This patient’s presentation is acute, with a clear onset following tonsillitis.
- B. Acute pyelonephritis – Unlikely.
- Acute pyelonephritis is an upper urinary tract infection that typically presents with fever, chills, and flank pain, not with the pink-red urine and edema seen in this case. Pyelonephritis also usually does not cause the level of proteinuria seen here.
- C. Hemorrhagic cystitis – Unlikely.
- Hemorrhagic cystitis typically presents with painful urination, hematuria, and bladder irritation, not with the systemic symptoms of headache, edema, and elevated blood pressure. The patient also has significant proteinuria, which is not typical of cystitis.
- E. Urolithiasis – Unlikely.
- Urolithiasis typically presents with flank pain, hematuria, and painful urination, but the absence of these symptoms and the presence of proteinuria, casts, and hypertension make this diagnosis less likely.
Conclusion:
The combination of a recent streptococcal infection, hematuria, proteinuria, and granular casts points to acute glomerulonephritis, a common complication following streptococcal tonsillitis. The elevated blood pressure and edema further support this diagnosis.
84. A 17-year-old patient complains of headache, sore throat, and fever. According to the patient’s medical history, the onset of the disease was 2 days ago. Objectively, the patient’s condition is severe, body temperature 38°C. The oropharyngeal mucosa is moderately hyperemic and edematous. FiImy deposits that are difficult to remove are observed on the tonsils, palate, and uvula. Palpation detects enlarged submandibular lymph nodes and edematous subcutaneous tissue on the neck. What is the most likely diagnosis in this case?
A. Peritonsillar abscess
B. Diphtheria
C. Acute lymphoblastic leukemia
D. Infectious mononucleosis
E. Tularemia
Correct Answer: B. Diphtheria
Explanation:
The patient presents with a sore throat, fever, headache, and the following signs on examination:
- Hyperemic and edematous oropharyngeal mucosa.
- Filmy deposits that are difficult to remove on the tonsils, palate, and uvula.
- Enlarged submandibular lymph nodes and edematous subcutaneous tissue on the neck.
These findings are highly suggestive of diphtheria, a bacterial infection caused by Corynebacterium diphtheriae. Diphtheria typically presents with:
- A thick, grayish-white membrane (diphtheritic membrane) in the throat, which is difficult to remove and can cover the tonsils, uvula, and palate.
- Enlarged lymph nodes (cervical lymphadenopathy, often referred to as “bull neck”).
- Systemic toxicity, including fever and headache.
The difficulty in removing the membrane and the associated systemic symptoms (fever, sore throat, headache) are characteristic of diphtheria, and the condition requires immediate medical attention due to the potential for airway obstruction and systemic complications.
Why Other Options Are Less Likely:
- A. Peritonsillar abscess – Unlikely.
- A peritonsillar abscess would typically present with severe unilateral tonsillar swelling, difficulty swallowing, trismus (jaw stiffness), and severe localized pain, rather than the widespread grayish membrane seen in this patient. The swelling and systemic symptoms are more characteristic of diphtheria.
- C. Acute lymphoblastic leukemia (ALL) – Unlikely.
- Although ALL can cause fever and lymphadenopathy, it would not cause the characteristic grayish membrane in the throat seen in this patient. Additionally, ALL usually presents with pallor, bruising, and bone pain, which are absent here.
- D. Infectious mononucleosis – Unlikely.
- While infectious mononucleosis can cause sore throat and lymphadenopathy, the hallmark of mononucleosis is exudative tonsillitis with a white or yellowish exudate, not the difficult-to-remove gray membrane seen in diphtheria. Moreover, mononucleosis usually causes splenomegaly, which is not mentioned in this case.
- E. Tularemia – Unlikely.
- Tularemia typically presents with ulceroglandular or glandular forms, which cause localized lymphadenopathy and ulceration, but it does not cause the thick, difficult-to-remove membrane in the throat, which is characteristic of diphtheria.
Conclusion:
Given the presentation of a grayish membrane in the throat, fever, headache, and enlarged cervical lymph nodes, the most likely diagnosis is diphtheria, and immediate treatment with antitoxin and antibiotics is required.
85 A 68-year-old man, undergoing inpatient treatment for unstable angina pectoris, suddenly lost his consciousness while walking and fell down. Objectively, there is no pulsation on the arterics, the pupils are constricted and unresponsive to light, rare low- amplitude movements of the chest (8-10/min) are observed, the blood pressure cannot be measured. ECG shows a sinusoidal curve with frequent waves, differing in shape and amplitude, with the rate of 350-400/min. What complication has developed in the patient?
A. Pulmonary thromboembolism
B. Asystole
C. Atrial fibrillation-
D. Complete atrioventricular block
E Ventricular fibrillation
Correct Answer: E. Ventricular fibrillation
Explanation:
The patient presents with the following signs:
- Sudden loss of consciousness and fall.
- No pulsation on the arteries.
- Constricted and unresponsive pupils (which suggests a very low perfusion state or brain hypoxia).
- Rare, low-amplitude chest movements (8-10/min), indicating ineffective breathing and likely inadequate circulation.
- Sinusoidal ECG with frequent waves, indicating irregular electrical activity in the ventricles, with a rate of 350-400/min.
These findings are consistent with ventricular fibrillation (VF), a life-threatening arrhythmia where the ventricles quiver ineffectively, and the heart cannot pump blood, leading to cardiac arrest. The sinusoidal ECG is characteristic of VF, which results in the loss of consciousness due to lack of blood flow to the brain and requires immediate defibrillation.
Why Other Options Are Less Likely:
- A. Pulmonary thromboembolism – Unlikely.
- Pulmonary embolism often presents with sudden dyspnea, chest pain, and hypoxia, but would not typically cause a sinusoidal ECG pattern. Furthermore, the loss of consciousness in this case is more likely due to cardiac arrhythmia than a sudden pulmonary event.
- B. Asystole – Unlikely.
- Asystole is characterized by no electrical activity on the ECG (a flat line), whereas the ECG in this case shows sinusoidal waves, which is consistent with ventricular fibrillation, not asystole. The patient’s symptoms are also more in line with VF.
- C. Atrial fibrillation – Unlikely.
- While atrial fibrillation causes an irregular heartbeat, it is usually much slower and is not associated with a sinusoidal waveform on the ECG. Also, atrial fibrillation would not explain the loss of consciousness and pulselessness seen in this case.
- D. Complete atrioventricular block – Unlikely.
- Complete AV block leads to slow, irregular ventricular rhythms, but would not produce the sinusoidal ECG pattern or the lack of pulses that is characteristic of ventricular fibrillation. It also usually causes bradycardia, rather than the rapid 350-400/min rate observed here.
Conclusion:
The sinusoidal ECG pattern, along with the patient’s unresponsiveness, absence of pulse, and poor chest movements, is indicative of ventricular fibrillation. This condition requires immediate defibrillation to restore normal rhythm and save the patient’s life.
86 An 8-year-old boy received a hit to the head With a ball during a physical training lesson. Over the next few days he was complaining of headache and nausea and had three episodes of vomiting. His parents did not take him to a physician. Six months later, the parents and the teachers began to notice that the boy started experiencing episodes, when his face would become red and he would freeze for a few seconds, focusing his gaze on a single point. While in this state, the boy was not responding to people calling his name. What is the most likely diagnosis in this case?
A Atonic absence seizure
B. Syncopal state
C. Atack of focal motor epilepsy
D Simple absence seizure
E. Vago-insular paroxysm
Correct Answer D. Simple absence seizure
Explanation:
The boy’s symptoms—episodes where his face becomes red, he freezes, focuses his gaze on a single point, and is unresponsive to stimuli—are characteristic of a simple absence seizure. Absence seizures are typically seen in childhood, especially after a head injury (as the boy had a hit to the head during a physical training lesson). The lack of movement (freezing), focus on a single point (gaze fixation), and unresponsiveness to external stimuli (not responding to name being called) point to an absence seizure.
These seizures usually last for a few seconds and are accompanied by a brief loss of awareness but no convulsions or motor symptoms. They are often non-convulsive and may be mistaken for daydreaming or zoning out, especially in children.
Why Other Options Are Less Likely:
- A. Atonic absence seizure – Unlikely.
- Atonic absence seizures typically involve a loss of muscle tone leading to sudden head dropping or collapsing, which doesn’t fit the described symptoms of the boy freezing and staring without any significant loss of muscle tone.
- B. Syncopal state – Unlikely.
- Syncope is typically characterized by fainting, loss of consciousness, and a brief episode of reduced blood flow to the brain, usually followed by recovery with some dizziness. The boy’s episodes don’t involve a loss of consciousness followed by recovery, and there’s no indication of dizziness, which would typically accompany a syncopal event.
- C. Attack of focal motor epilepsy – Unlikely.
- Focal motor epilepsy would involve motor symptoms like muscle jerking or movements that start in one area of the body. The boy’s episodes do not involve motor symptoms; they are described as a staring episode without any involuntary muscle movement.
- E. Vago-insular paroxysm – Unlikely.
- Vago-insular paroxysms are associated with episodes of vagal syncope due to parasympathetic overactivity, usually triggered by stress or pain. These episodes typically involve brief loss of consciousness due to sudden drops in heart rate or blood pressure. However, the described episodes of staring and no response to stimuli are more typical of an absence seizure, not vagal syncope.
Conclusion:
The boy’s history of a head injury followed by episodes of staring, gaze fixation, and lack of response points to a diagnosis of simple absence seizures, which are common in children and can sometimes appear after a head injury. These seizures are non-convulsive and brief, involving loss of awareness without significant motor symptoms.
87. A 6-year-old child complains of a headache, weakness, and pain during chewing. Objectively, bilateral enlargement of the salivary glands is observed. The enlarged glands fill the retromandibular fossa. The skin over the glands is tense, glossy, and normal-colored. Body temperature 39°C. The oral mucosa is dry, the external opening of the salivary gland duct is edematous. What is the most likely diagnosis in this case?
A. Infectious mononucleosis
B. Epidemic parotitis
C. Tumors of the salivary glands
D. Diphtheria
E. Sialolithiasis
Correct Answer: B. Epidemic parotitis
Explanation:
The child’s symptoms—headache, weakness, pain during chewing, bilateral enlargement of the salivary glands, skin over the glands being tense and glossy, dry oral mucosa, and fever—are characteristic of epidemic parotitis (mumps). This viral infection primarily affects the parotid glands, which are the major salivary glands located near the ears.
Key features supporting the diagnosis:
- Bilateral enlargement of the salivary glands, especially the parotid glands, which is a hallmark of mumps.
- Pain during chewing (due to inflammation of the salivary glands).
- Dry oral mucosa and edematous external opening of the salivary gland duct due to duct obstruction from inflammation.
- Fever (39°C), which is common in viral infections like mumps.
Why Other Options Are Less Likely:
- A. Infectious mononucleosis – Unlikely.
- Infectious mononucleosis is caused by the Epstein-Barr virus and typically presents with sore throat, enlarged lymph nodes, splenomegaly, and fatigue. While it can cause mild swelling of the salivary glands, the bilateral parotid enlargement seen here is more typical of mumps.
- C. Tumors of the salivary glands – Unlikely.
- Tumors of the salivary glands tend to cause asymmetrical swelling and are usually non-tender. The child’s pain, fever, and acute presentation with bilateral gland enlargement point to an infectious cause rather than a neoplastic one.
- D. Diphtheria – Unlikely.
- Diphtheria is a bacterial infection that typically causes a membranous pharyngitis with sore throat, fever, and pseudomembranes in the throat. While it can cause neck swelling due to lymphadenopathy, it does not cause bilateral salivary gland enlargement or pain during chewing as seen in this case.
- E. Sialolithiasis – Unlikely.
- Sialolithiasis (salivary stones) typically causes unilateral swelling of the affected gland, often with pain or swelling that worsens when eating. The bilateral involvement, fever, and overall presentation point away from sialolithiasis, which would be much more localized.
Conclusion:
The most likely diagnosis is epidemic parotitis (mumps), a viral infection that causes bilateral swelling of the parotid glands, fever, and pain during chewing, which is characteristic in children.
88. On the second day after a total thyroidectomy, a 56-year-old woman developed severe agitation, fear of death, hand tremor, nausea, vomiting, diarrhea, and fever. Objectively, the following is observed: hot and moist skin, profuse sweating, acute muscle weakness, adynamia, dry tongue and lips, body temperature 40°C. Heart rate increases from 60-70/min to 130-220/min. Blood pressure -210/70 mm Hg. Auscultation detects intensified and arrhythmic heart sounds, systolic murmur can be heard at the apex and in the Botkin-Erb’s point. ECG shows a non-sinusoidal irregular rhythm and atrial fibrillation, the ventricular contraction rate is 140-160- 220/min. What is the most likely diagnosis in this case?
A. Septic shock
B. Hypertensive crisis
C. Acute cardiovascular failure
D. Food poisoning
E Thyrotoxic crisis
Correct Answer: E. Thyrotoxic crisis
Explanation:
The patient’s symptoms—severe agitation, fear of death, hand tremor, nausea, vomiting, diarrhea, fever, sweating, acute muscle weakness, and a sudden onset of tachycardia (130-220/min), fever (40°C), and atrial fibrillation—are highly suggestive of thyrotoxic crisis (thyroid storm).
Key features supporting this diagnosis:
- Post-thyroidectomy: The patient has recently undergone total thyroidectomy, and thyroid storm can occur due to sudden release of thyroid hormones following surgery or as a complication of thyroid disease.
- Hyperthyroid symptoms: The patient is experiencing classic symptoms of hyperthyroidism, such as agitation, heat intolerance (hot and moist skin), tremor, and sweating.
- Cardiac signs: The tachycardia, atrial fibrillation, and systolic murmur (likely secondary to the hyperdynamic circulation associated with thyrotoxicosis) are consistent with thyroid storm.
- Fever and gastrointestinal symptoms: The patient presents with a high fever (40°C), nausea, vomiting, and diarrhea, all of which are typical features of a thyroid storm.
Why Other Options Are Less Likely:
- A. Septic shock – Unlikely.
- While the patient has fever, septic shock would typically present with hypotension, tachypnea, and evidence of infection, which are not present here. The patient’s high blood pressure and absence of infection make septic shock less likely.
- B. Hypertensive crisis – Unlikely.
- Although the patient has high blood pressure, the associated fever, tachycardia, atrial fibrillation, and neurological symptoms point to a thyroid storm rather than a hypertensive crisis. Hypertensive crisis typically does not cause the hyperthermia, gastrointestinal symptoms, or mental confusion seen here.
- C. Acute cardiovascular failure – Unlikely.
- Acute cardiovascular failure would typically cause hypotension and signs of shock, which are not seen here. The patient is hypertensive and has tachycardia, which suggests thyrotoxicosis rather than heart failure.
- D. Food poisoning – Unlikely.
- Food poisoning typically presents with gastrointestinal symptoms (nausea, vomiting, diarrhea) and may be associated with fever, but it would not cause the neurological symptoms, tachycardia, atrial fibrillation, and elevated blood pressure seen in this patient. The history of thyroidectomy makes thyroid storm a more likely diagnosis.
Conclusion:
The most likely diagnosis is thyrotoxic crisis (thyroid storm), a life-threatening condition that can occur after thyroid surgery, characterized by fever, tachycardia, mental agitation, and gastrointestinal symptoms. Immediate treatment is required to stabilize the patient and manage the excess thyroid hormone levels.
89. A 28-year-old woman has been hospitalized with complaints of dizziness, bloody discharge from her genital tract, and pain that suddenly appeared in her lower abdomen and radiates into the anal region. According to the patient’s medical history, the delay of menstruation is 4 weeks. Signs of peritoneal irritation are positive. During bimanual examination, the uterus and its appendages are sharply painful and cannot be clearly outlined. “Fluctuating uterus”sign and protrusion and tenderness of the posterior formix are observed. Body temperature 36.7°C. Blood test results: hemoglobin 82 g/L, erythrocytes -2.8-1012/L. What is the most likely diagnosis in this case?
A. Torsion of the pedicle of an ovarian tumor
B. Acute appendicitis
C. Acute right-sided adnexitis
D. Interrupted ectopic pregnancy
E. Ovarian apoplexy
Correct Answer: D. Interrupted ectopic pregnancy
Explanation:
The patient presents with a 4-week delay in menstruation, dizziness, bloody discharge, and lower abdominal pain radiating to the anal region, all of which are indicative of an ectopic pregnancy. The additional findings, such as signs of peritoneal irritation, tenderness of the uterus and appendages, and the “fluctuating uterus” sign, suggest that there has been a rupture or interruption of the ectopic pregnancy.
Key features supporting this diagnosis:
- Delay in menstruation: A delay of 4 weeks in menstruation raises suspicion for a pregnancy, and in this context, it is likely an ectopic pregnancy (a pregnancy occurring outside the uterus, often in the fallopian tube).
- Abdominal pain: The sharp, sudden onset of lower abdominal pain, especially radiating to the anal region, is common in cases of ruptured or interrupted ectopic pregnancy. This pain typically occurs due to bleeding into the peritoneum.
- Positive signs of peritoneal irritation: These are highly suggestive of a ruptured ectopic pregnancy, as peritoneal irritation occurs when blood or fluid leaks into the peritoneal cavity, often from a ruptured ectopic pregnancy.
- Fluctuating uterus sign and tenderness in the posterior fornix: These findings point toward a ruptured ectopic pregnancy or hemoperitoneum (blood in the peritoneal cavity), which can occur after the rupture of the fallopian tube.
- Anemia: The patient’s low hemoglobin (82 g/L) suggests blood loss, which is common with ruptured ectopic pregnancy due to internal bleeding.
Why Other Options Are Less Likely:
- A. Torsion of the pedicle of an ovarian tumor – Unlikely.
- Ovarian torsion typically presents with severe, sudden-onset unilateral lower abdominal pain. While it can cause nausea and vomiting, it usually doesn’t cause bloody discharge or peritoneal irritation unless there is necrosis or rupture of the ovary. This patient’s presentation is more consistent with a ruptured ectopic pregnancy.
- B. Acute appendicitis – Unlikely.
- Although acute appendicitis can cause abdominal pain, it does not typically present with bloody discharge or the characteristic signs of a ruptured ectopic pregnancy (e.g., peritoneal irritation and fluctuating uterus). The absence of right lower quadrant pain and the delay in menstruation makes this diagnosis less likely.
- C. Acute right-sided adnexitis – Unlikely.
- Acute adnexitis (pelvic inflammatory disease) can cause lower abdominal pain and tenderness of the uterus and appendages, but it would typically not cause dizziness or signs of peritoneal irritation associated with a ruptured ectopic pregnancy. Additionally, there would likely be signs of infection (e.g., fever, elevated white blood cells), which are not observed here.
- E. Ovarian apoplexy – Unlikely.
- Ovarian apoplexy (hemorrhage into the ovary) can cause acute abdominal pain and peritoneal irritation, but it typically presents with unilateral lower abdominal pain and is not typically associated with a delay in menstruation or bloody discharge, as in the case of a ruptured ectopic pregnancy.
Conclusion:
The most likely diagnosis is interrupted ectopic pregnancy. The combination of a delayed menstruation, abdominal pain, bloody discharge, and signs of peritoneal irritation strongly suggests a ruptured ectopic pregnancy, which requires urgent surgical intervention.
90. A case of occupational poisoning is being investigated, where an auto mechanic, who was testing a diesel engine in a garage with damaged exhaust ventilation, developed the following symptoms at the end of the working day: headache, tinnitus, nausea, vomiting, and pulse lability. Objectively, his skin and mucosa are colored cherry-red. What toxic agent has caused the acute poisoning in this worker?
A. Carbon disulfide
B. Carbon monoxide
C. Carbon dioxide
D. Sulfur dioxide
E. Nitrous oxide
Correct Answer: B. Carbon monoxide
Explanation:
The patient’s symptoms of headache, tinnitus, nausea, vomiting, and pulse lability, combined with the cherry-red coloring of the skin and mucosa, are classic signs of carbon monoxide (CO) poisoning.
Key signs of carbon monoxide poisoning:
- Cherry-red skin and mucosa: One of the hallmark signs of carbon monoxide poisoning is the cherry-red coloration of the skin and mucous membranes due to the binding of carbon monoxide to hemoglobin, forming carboxyhemoglobin. This prevents proper oxygen transport, leading to hypoxia and the characteristic color change.
- Symptoms of CO poisoning: Headache, dizziness, nausea, vomiting, confusion, and pulse lability are all consistent with carbon monoxide poisoning.
- Occupational exposure: This mechanic works in a garage with damaged exhaust ventilation, increasing the risk of carbon monoxide exposure, which is common in poorly ventilated areas where internal combustion engines (such as diesel engines) are run.
Why Other Options Are Less Likely:
- A. Carbon disulfide – Unlikely.
- Carbon disulfide exposure typically leads to symptoms like neurological and cardiovascular changes, such as dizziness, headaches, and mood disturbances. It does not cause cherry-red skin or mucosa, which is characteristic of carbon monoxide poisoning.
- C. Carbon dioxide – Unlikely.
- Carbon dioxide (CO2) poisoning is typically associated with symptoms of respiratory distress, tachypnea, and hypercapnia. It does not cause cherry-red skin or mucosa, and it is more likely to occur in situations of poor ventilation but without the distinctive signs seen in CO poisoning.
- D. Sulfur dioxide – Unlikely.
- Sulfur dioxide exposure primarily causes respiratory symptoms such as coughing, wheezing, and shortness of breath, and is not associated with cherry-red skin. The symptoms described here are more consistent with carbon monoxide exposure.
- E. Nitrous oxide – Unlikely.
- Nitrous oxide poisoning can cause symptoms such as dizziness, nausea, and confusion, but it does not cause cherry-red skin and mucosa. Nitrous oxide poisoning is less common in occupational settings compared to carbon monoxide, especially with poor ventilation around internal combustion engines.
Conclusion:
The most likely toxic agent causing the acute poisoning in this worker is carbon monoxide (CO). The symptoms and the characteristic cherry-red skin color point directly to CO exposure, commonly associated with incomplete combustion in poorly ventilated spaces. Immediate treatment typically includes oxygen therapy to displace carbon monoxide from hemoglobin and prevent further toxicity.
91. A 43-year-old patient complains of watery diarrhea that occurs up to 10 times a day, cramping abdominal pain, and fever. The patient has history of recent antibi- otic therapy (clindamycin and tobramycin) for pancreonecrosis. Blood test detects leukocytosis. What is the most likely diagnosis, in this case?
A. Interloop abscess
B. Toxic-megacolon
C-Pseudomembranous colitis
D. Crohn’s disease
E. Shigellosis
Correct Answer: C. Pseudomembranous colitis
Explanation:
The patient’s history of recent antibiotic therapy (specifically clindamycin and tobramycin) and the subsequent onset of watery diarrhea, cramping abdominal pain, and fever, along with the presence of leukocytosis, strongly suggests pseudomembranous colitis.
Pseudomembranous colitis is primarily caused by an overgrowth of Clostridium difficile (C. difficile) following antibiotic treatment, which disrupts the normal gut flora, allowing the pathogenic bacteria to proliferate.
Key features of pseudomembranous colitis:
- Recent antibiotic therapy: Especially clindamycin, which is a well-known risk factor for C. difficile infection.
- Diarrhea: Typically watery and can occur frequently (up to 10 times per day).
- Abdominal pain and fever: Cramping abdominal pain and fever are common symptoms.
- Leukocytosis: An elevated white blood cell count is often seen in the infection.
Diagnosis: Confirmed by the detection of C. difficile toxin in stool samples or colonoscopy showing pseudomembranous lesions in the colon.
Why Other Options Are Less Likely:
- A. Interloop abscess – Unlikely.
- An interloop abscess typically occurs due to complicated intra-abdominal infections such as diverticulitis or pancreatitis, and is usually characterized by localized pain, fever, and signs of infection. The history and symptoms here point more toward a colitis-related condition, not an abscess.
- B. Toxic-megacolon – Less likely.
- Toxic-megacolon is a serious complication of ulcerative colitis or C. difficile infection. While it can occur in pseudomembranous colitis, it is typically associated with severe cases, often with extreme abdominal distension and systemic toxicity (e.g., sepsis). This patient doesn’t show the characteristic signs of toxic-megacolon.
- D. Crohn’s disease – Less likely.
- Crohn’s disease typically presents with chronic symptoms of abdominal pain, diarrhea, and weight loss over a longer duration. The patient’s recent antibiotic use and the acute onset of symptoms are more indicative of C. difficile infection, which is not characteristic of Crohn’s.
- E. Shigellosis – Unlikely.
- Shigellosis causes bloody diarrhea, cramping abdominal pain, and fever, but there is no indication of bloody stools in this patient. Additionally, recent antibiotic therapy is more likely to result in C. difficile infection, which causes watery diarrhea, rather than a Shigella infection.
Conclusion:
The most likely diagnosis is pseudomembranous colitis, caused by Clostridium difficile after recent antibiotic therapy. Treatment typically involves discontinuing the causative antibiotics and initiating therapy with oral vancomycin or fidaxomicin, and in severe cases, intravenous metronidazole may be used.
92. A 52-year-old patient complains of mixed-type dyspnea and chronic cough wi- th expectoration of a large amount of foul- smelling mucopurulent sputum, sometimes with blood, that occurs mostly in the morning as a “full mouth”. He has history of frequent exacerbations of chronic bronchi- tis. Objectively, he has acrocyanosis and clubbed “drumstick fingers. Respiratory rate 22/min. Auscultation detects harsh breathing, fine and medium bubbling crackles can be heard in the lower lung segments on both si- des. Spiral computed tomography of of the chest detects areas of carnification, thin-walled cavities, and cylindrical dilatations of the bronchi. What is the most likely diagnosis in this case?
A. Chronic obstructive pulmonary disease
B. Bronchiectasis
C. Bronchial asthma
D. Community-acquired pneumonia
E. Pulmonary abscess
Correct Answer: B. Bronchiectasis
Explanation:
The patient’s history and symptoms suggest bronchiectasis, a condition characterized by chronic, irreversible dilation of the bronchi, and associated with chronic cough, mucopurulent sputum, dyspnea, and recurrent respiratory infections.
Key features pointing to bronchiectasis:
- Chronic cough with foul-smelling mucopurulent sputum, sometimes with blood, especially in the morning. This is a classic presentation for bronchiectasis, where the dilated bronchi become sites for the accumulation of secretions.
- History of frequent exacerbations of chronic bronchitis, which can lead to bronchiectasis over time.
- Acrocyanosis and clubbing of the fingers are signs of chronic hypoxia, which can occur in bronchiectasis due to poor oxygen exchange.
- Harsh breathing and bubbling crackles heard on auscultation suggest chronic infection and secretions in the lungs, which is consistent with bronchiectasis.
- Spiral computed tomography (CT) findings of carnification (fibrosis), thin-walled cavities, and cylindrical dilatation of the bronchi are diagnostic features of bronchiectasis.
Why Other Options Are Less Likely:
- A. Chronic obstructive pulmonary disease (COPD) – Unlikely.
- While COPD can cause chronic cough and dyspnea, it usually presents with progressive airflow limitation and emphysema rather than the localized bronchial dilation seen in bronchiectasis. Additionally, the foul-smelling sputum and cavitary lesions are more characteristic of bronchiectasis than COPD.
- C. Bronchial asthma – Unlikely.
- Asthma typically presents with episodic wheezing, shortness of breath, and reversible airflow obstruction. It does not cause chronic cough with purulent sputum or cylindrical dilatation of the bronchi seen on CT. Also, asthma does not cause the drumstick fingers or acrocyanosis seen in this case.
- D. Community-acquired pneumonia – Unlikely.
- Pneumonia usually presents acutely with fever, chills, chest pain, and productive cough. This patient’s chronic symptoms, including cough, dyspnea, and long-standing sputum production, are more consistent with bronchiectasis.
- E. Pulmonary abscess – Unlikely.
- Pulmonary abscesses are usually associated with foul-smelling sputum, but they tend to be more localized and acute in onset, with fever, weight loss, and a more isolated area of infection rather than the widespread bronchial dilation seen in bronchiectasis.
Conclusion:
The most likely diagnosis is bronchiectasis, characterized by chronic cough, purulent sputum, dyspnea, and findings of cylindrical bronchial dilatation on CT. The management typically includes antibiotics for acute exacerbations, bronchodilators, and chest physiotherapy to improve mucus clearance.
93. Examination of a one-month-old chi- Id detects microphthalmia, microcephaly, hypotclorism, bilateral cleft lip and palate, polydactyly, and a ventricular septal defect. Genetic studies revealed trisomy 13. What is the most likely diagnosis in this case?
A. Roberts syndrome
B. Patau syndrome
C. Lejeune syndrome
D. Arnold-Chiari syndrome
E. Down syndrome
Correct Answer: B. Patau syndrome
Explanation:
The patient presents with microphthalmia, microcephaly, hypotelorism, bilateral cleft lip and palate, polydactyly, and a ventricular septal defect. These features are characteristic of Patau syndrome, which is also known as trisomy 13.
Key Features of Patau Syndrome (Trisomy 13):
- Microphthalmia (small eyes) and microcephaly (small head).
- Hypotelorism (closely spaced eyes).
- Bilateral cleft lip and palate.
- Polydactyly (extra fingers or toes).
- Ventricular septal defect (a hole in the heart).
- Trisomy 13 is confirmed through genetic studies (as mentioned in the case) and is associated with a poor prognosis, with many affected infants dying in infancy or early childhood.
Why Other Options Are Less Likely:
- A. Roberts syndrome – Unlikely.
- Roberts syndrome is a rare genetic disorder characterized by limb abnormalities, craniofacial defects, and growth retardation, but it does not typically include the combination of features seen in this patient, such as polydactyly and ventricular septal defect.
- C. Lejeune syndrome – Unlikely.
- Lejeune syndrome, or Cri-du-chat syndrome, is associated with a missing portion of chromosome 5 and is characterized by a high-pitched cry, microcephaly, and developmental delay, but it does not feature the polydactyly or ventricular septal defect seen in this case.
- D. Arnold-Chiari syndrome – Unlikely.
- Arnold-Chiari syndrome primarily involves cerebellar tonsil herniation and can be associated with hydrocephalus and spinal cord defects, but it does not present with the facial abnormalities, polydactyly, or heart defects seen here.
- E. Down syndrome – Unlikely.
- Down syndrome (trisomy 21) is characterized by distinctive facial features (e.g., epicanthal folds, flat nasal bridge), intellectual disability, and congenital heart defects, but Patau syndrome (trisomy 13) better explains the combination of microphthalmia, cleft lip/palate, polydactyly, and ventricular septal defect.
Conclusion:
The most likely diagnosis is Patau syndrome (trisomy 13), which presents with microphthalmia, microcephaly, hypotelorism, cleft lip and palate, polydactyly, and ventricular septal defect, among other features. Genetic testing confirms trisomy 13.
94. A 32-year-old patient complains of decreased visual acuity, nausea, iridescent circles in the eyes when looking at the light, and pain in the right eye. The pain radiates to the right temple. Objectively, the following is observed: right eye stagnant injection of scleral vessels, edematous cornea, shallow anterior chamber, dilated pupil. The fundus reflex is dull pink because of corneal edema. Intraocular – -58 mm Hg. What is the most likely diagnosis in this case?
A. Swelling cataract
B. Open-angle glaucoma
C. Bacterial conjunctivitis
D. Iridocyclitis
E Acute attack of glaucoma
Correct Answer: E. Acute attack of glaucoma
Explanation:
The patient presents with decreased visual acuity, nausea, iridescent circles around light (a sign of halos), pain in the right eye radiating to the temple, and clinical findings including stagnant injection of scleral vessels, edematous cornea, shallow anterior chamber, dilated pupil, dull pink fundus reflex, and an elevated intraocular pressure of 58 mm Hg. These signs and symptoms are characteristic of an acute attack of glaucoma, specifically angle-closure glaucoma.
Key Features of Acute Angle-Closure Glaucoma:
- Severe eye pain, often radiating to the temple or head.
- Nausea and vomiting, which can be related to the increased intraocular pressure.
- Halos or iridescent circles around lights due to corneal edema.
- Stagnant scleral vessels and a dilated pupil that does not react well to light.
- Edematous cornea and shallow anterior chamber.
- Markedly elevated intraocular pressure (in this case, 58 mm Hg), which can be life-threatening if untreated.
Acute angle-closure glaucoma is a medical emergency, and treatment should be initiated immediately to lower intraocular pressure to prevent permanent vision loss.
Why Other Options Are Less Likely:
- A. Swelling cataract – Unlikely.
- Cataracts cause gradual vision loss and blurring, but they do not present with pain, nausea, or elevated intraocular pressure. Additionally, the shallow anterior chamber and dilated pupil are not characteristic of cataracts.
- B. Open-angle glaucoma – Unlikely.
- Open-angle glaucoma typically causes gradual, painless vision loss without acute symptoms or markedly elevated intraocular pressure. The patient’s symptoms and the very high intraocular pressure point to an acute attack rather than open-angle glaucoma.
- C. Bacterial conjunctivitis – Unlikely.
- Bacterial conjunctivitis typically presents with redness, discharge, and irritation but does not cause pain, nausea, elevated intraocular pressure, or corneal edema. It also does not involve the shallow anterior chamber seen in this patient.
- D. Iridocyclitis – Unlikely.
- Iridocyclitis (anterior uveitis) can cause eye pain, redness, and light sensitivity, but it usually does not lead to such high intraocular pressure. The shallow anterior chamber and dilated pupil are more suggestive of glaucoma than uveitis.
Conclusion:
The most likely diagnosis is acute attack of glaucoma (specifically angle-closure glaucoma), which is a medical emergency requiring immediate treatment to reduce intraocular pressure and prevent permanent vision loss..
95. During a routine medical examination of a 14-year-old boy, the following data were obtained: body length 142 cm (+2 sigma), body weight 36 kg (+1.5 sigma), chest ci- rcumference74 cm (+0.2 sigma). Make an assessment of the harmoniousness of the Id’s physical development.
A. Disharmonious
B. Excessive
C. Harmonious
D. Severely disharmonious
E. Above average
Correct Answer: C. Harmonious
Explanation:
The physical development of the boy is assessed based on standard deviations (sigma) from the average for his age group. Here are the key measurements and their interpretation:
- Body length: 142 cm (+2 sigma) – This means the boy’s height is above the average for his age group by 2 standard deviations, which is a sign of taller stature.
- Body weight: 36 kg (+1.5 sigma) – The boy’s weight is also above the average for his age group by 1.5 standard deviations, indicating that he is somewhat heavier than the average.
- Chest circumference: 74 cm (+0.2 sigma) – The chest circumference is just slightly above the average for his age group, showing a modest increase.
All three measurements are above the average, but they are proportionately higher and do not show significant discrepancy between each other. Since there is no major difference or imbalance between these growth parameters, the physical development is considered harmonious.
Why Other Options Are Less Likely:
- A. Disharmonious – This would be the case if there was a significant imbalance or discrepancy between the body measurements, such as extreme differences in height, weight, and chest circumference. Here, the measurements are relatively balanced.
- B. Excessive – This would imply that the boy’s growth measurements are excessively high compared to the norm for his age. While his measurements are above average, they are not excessively so, thus “excessive” is not the right term.
- D. Severely disharmonious – For this term to apply, there would need to be large, abnormal discrepancies in the physical development, which is not the case here.
- E. Above average – While the boy’s measurements are above average, “harmonious” better describes the overall balance of his growth as opposed to just categorizing him as “above average.”
Conclusion:
The boy’s physical development is harmonious based on the fact that his height, weight, and chest circumference are all above the average, but there is no major imbalance between them.
96. A 15-year-old girl complains of enlarged cervical lymph nodes and dry unproductive cough. According to the patient’s medi- cal history, the disease onset was 6 months ago. Objectively, enlarged lymph nodes form a conglomerate over the left clavicle, they are painless and not fused with the surrounding tissues, the skin above them is unchanged. Other groups of lymph nodes are normal. Auscultation detects vesicular breathing over the lungs, there are no wheezes. The liver protrudes 3 cm from under the edge of the costal arch, the spleen protrudes 2 cm. No pathology has been detected in other organs and systems. What is the most likely diagnosis in this case?
A. Lymphogranulomatosis
B. Tuberculosis
C. Acute lymphadenitis
D. Aeute lymphoblastic leukemia
E infectious mononucleosis
Correct Answer: A. Lymphogranulomatosis (Hodgkin lymphoma)
Explanation:
This patient presents with several key features that suggest Hodgkin lymphoma (also known as lymphogranulomatosis), a type of cancer affecting the lymphatic system.
Key points for this diagnosis:
- Enlarged, painless cervical lymph nodes: Painless, non-tender, and movable lymph nodes that have formed a conglomerate, particularly in the cervical region, are characteristic of Hodgkin lymphoma.
- Prolonged duration: The 6-month duration of symptoms, along with the slow progression, is common for Hodgkin lymphoma.
- Splenomegaly and hepatomegaly: Mild enlargement of the liver (3 cm below the costal margin) and spleen (2 cm) can be seen in Hodgkin lymphoma as part of the systemic involvement.
- Dry unproductive cough: This symptom may be related to enlarged lymph nodes near the lungs or trachea, causing pressure on the airways.
- Normal auscultation findings: No wheezes or rales suggests that the lungs themselves are not primarily involved, but rather the mediastinal or hilar lymph nodes could be enlarged.
In Hodgkin lymphoma, the enlargement of lymph nodes often occurs in the cervical, supraclavicular, or axillary regions, and the presence of systemic symptoms like fever, night sweats, and weight loss may also be present.
Why Other Options Are Less Likely:
- B. Tuberculosis:
- While tuberculosis can cause enlarged lymph nodes, it is more often associated with systemic symptoms like fever, weight loss, night sweats, and a productive cough (not dry). The prolonged, painless nature of the lymph node enlargement and absence of pulmonary symptoms make tuberculosis less likely.
- C. Acute lymphadenitis:
- Acute lymphadenitis is usually associated with a bacterial infection and is often painful, with redness and swelling in the affected area. The absence of pain and the chronicity of symptoms make acute lymphadenitis less likely.
- D. Acute lymphoblastic leukemia:
- While lymphadenopathy can occur in leukemia, it is usually associated with other signs like pallor, bruising, or petechiae due to bone marrow involvement. Additionally, there would be signs of systemic illness like fatigue and bleeding, which are not noted in this case.
- E. Infectious mononucleosis:
- Infectious mononucleosis caused by the Epstein-Barr virus (EBV) often presents with more prominent throat symptoms (sore throat, tonsillar enlargement), fever, and significant fatigue. Lymphadenopathy can be present but would generally be more generalized, including the axillary and inguinal nodes. The lack of prominent fever and throat issues makes this less likely.
Conclusion:
The most likely diagnosis is lymphogranulomatosis (Hodgkin lymphoma) based on the chronic, painless cervical lymphadenopathy, mild splenomegaly and hepatomegaly, and dry cough without significant pulmonary involvement.
97 A 72-year-old patient complains of a rash, intense pain in her right subscapular region, general weakness, and fever of 377°C. According to the patient’s medical history, the disease onset was three days ago. Objectively, grouped vesicles with transparent contents, arranged in a line, can be detected in the right subscapular region on hyperemic, slightly infiItrated skin. What is the most likely diagnosis in this case?
A. Allergic dermatitis
B. Duhring’s dermatitis
C. Erysipelas
D. Herpes zoster
E. Impetigo
Correct Answer: D. Herpes zoster
Explanation:
This patient presents with a grouped vesicular rash, which is arranged in a line along with associated symptoms such as pain, fever, and general weakness. These findings are most consistent with herpes zoster (shingles).
Key features supporting this diagnosis:
- Painful rash: Herpes zoster is commonly associated with a painful, burning sensation before the appearance of the rash.
- Grouped vesicles on hyperemic, slightly infiltrated skin: The rash in herpes zoster typically starts as small, grouped vesicles that follow a dermatomal distribution, usually on one side of the body.
- Location: The right subscapular region is consistent with the dermatome affected by the shingles virus (the T3-T6 dermatomes, which affect areas of the chest and back).
- Fever and general weakness: These systemic symptoms often accompany a herpes zoster infection, especially in older adults.
Herpes zoster occurs due to the reactivation of the varicella-zoster virus (the same virus that causes chickenpox), often triggered by a weakened immune system or aging.
Why Other Options Are Less Likely:
- A. Allergic dermatitis:
- Allergic dermatitis typically presents with itching, redness, and irritation but is not usually associated with the grouped vesicular rash and pain seen in this patient. The pattern of distribution and vesicle formation is not characteristic of allergic dermatitis.
- B. Duhring’s dermatitis:
- Duhring’s dermatitis herpetiformis (associated with celiac disease) presents as intensely pruritic papulovesicular lesions, but they are symmetrical, not unilaterally grouped along a dermatome as seen in herpes zoster.
- C. Erysipelas:
- Erysipelas is a bacterial skin infection caused by group A streptococcus. It presents with well-demarcated red, swollen, and painful areas of skin, typically on the face or legs, with systemic signs of infection (fever, chills). It does not cause grouped vesicles along a dermatomal line.
- E. Impetigo:
- Impetigo is a superficial bacterial skin infection caused by staphylococcus or streptococcus. It leads to honey-colored crusted lesions, rather than vesicles. It also lacks the unilateral, dermatomal distribution seen in herpes zoster.
Conclusion:
The most likely diagnosis is herpes zoster (shingles), which is characterized by a grouped vesicular rash along a dermatome, often accompanied by pain, fever, and weakness.
98. A 23-year-old patient complains of edema and pain in the knee and elbow joints, rapid heart rate, and fever of 38°C. According to the patient’s medical history, 2 weeks ago he had a case of an acute respiratory viral infection. Objectively, edema of knee and elbow joints is observed. Pulse 94/min, transverse cardi- ac diameter 14 cm. Auscultation detects weakening of the first heart sound and a systolic murmur at the apex. Complete blood count: leukocytes 9.1 109/L, ESR 18 mm/hour. Urinalysis: protein 0.033 g/L, leukocytes 6-8 in sight. ECG shows sinus tachycardia, PQ interval – 0.24 seconds. What is the most likely diagnosis in this case?
A. Reiter’s disease
B. Acute rheumatic fever
C. Gout
D. Osteoarthritis
E. Rheumatoid arthritis
Correct Answer: B. Acute rheumatic fever
Explanation:
The patient’s symptoms, including joint pain and swelling (especially in the knees and elbows), fever, and a history of a preceding upper respiratory infection, suggest acute rheumatic fever (ARF). The presence of a systolic murmur, signs of mitral valve involvement (weakened first heart sound), and prolonged PQ interval (suggesting heart conduction issues) are characteristic of ARF. This condition typically follows a streptococcal throat infection and can involve multiple systems, including the heart (carditis), joints (migratory arthritis), and skin.
Why Other Options Are Incorrect:
E. Rheumatoid arthritis: Rheumatoid arthritis presents with symmetrical joint involvement, morning stiffness, and often affects smaller joints. This patient’s presentation, following a recent viral infection and the involvement of larger joints (knee, elbow), makes rheumatoid arthritis less likely..
A. Reiter’s disease (Reactive arthritis): While Reiter’s disease can present with joint involvement following an infection, it typically follows a gastrointestinal or urogenital infection, not an upper respiratory infection. The absence of urethritis or conjunctivitis makes it less likely.
C. Gout: Gout typically presents with severe pain, redness, and swelling in one joint (usually the big toe) and is not associated with the systemic symptoms seen in this patient. It is caused by the deposition of uric acid crystals in joints.
D. Osteoarthritis: Osteoarthritis generally develops slowly with joint pain and stiffness, primarily in weight-bearing joints. It is not typically associated with fever or the other systemic signs seen in this patient.
99. A 45-year-old woman complains of constant pain in her right iliac region and frequent urination. According to the patient’s history, the disease onset was 12 hours ago. First she felt nausea and pain in her epigastric region. Five hours later the pain moved into the right iliac region. Objectively, the patient’s condition is moderately severe, she lies on her right side with the legs pulled up to the stomach, body temperature 37.9°C. The abdomen is symmetrical and not distended, during palpation there is tension and pain in the right iliac region. Rovsing, Blumberg, and Rosenstein (Sitkovsky) signs are positive. Rectal examination detects pain when pressing on the front wall of the rectum. What is the most likely diagnosis in this case?
A. Acute appendicitis
B. Acute cystitis
C. Acute adnexitis
D. Appendicular abscess
E. Appendicular infiltrate
Correct Answer: A. Acute appendicitis
Explanation:
The patient presents with classic signs of acute appendicitis:
- Pain migration from the epigastric region to the right iliac fossa (characteristic of appendicitis).
- Positive physical signs like Rovsing’s, Blumberg’s, and Rosenstein (Sitkovsky), which are indicative of peritoneal irritation from an inflamed appendix.
- Nausea, vomiting, and fever also support this diagnosis.
- Tenderness in the right iliac region and pain on rectal examination further suggest inflammation in the right lower abdomen, where the appendix is located.
Why Other Options Are Incorrect:
E. Appendicular infiltrate: This can occur in the aftermath of an inflamed appendix but usually involves more chronic signs such as a palpable mass and is not typically associated with the acute symptoms of pain migration and fever observed here.
B. Acute cystitis: This would typically present with lower abdominal pain, dysuria, and increased frequency of urination. However, the pain migration, fever, and abdominal signs in this patient point more toward appendicitis.
C. Acute adnexitis (pelvic inflammatory disease): This would present with lower abdominal pain and signs of pelvic tenderness, but the pain in this case is more localized to the right iliac region, which is not typical for adnexitis.
D. Appendicular abscess: This condition can result from untreated acute appendicitis but would present with a more prolonged course and additional symptoms of localized swelling or mass formation, which are not described here.
100. An 18-year-old patient complains of pain in her lower abdomen, fever of 375°C, a significant amount of mucopurulent discharge being produced from her genital tract, and burning pain during urination. Mirror examination and vaginal examination reveal an infiltrated urethra and hyperemic and eroded cervix. The uterus is painful to palpation, the appendages are painful and thickened, the fornices are free. Bacterioscopy of the discharge detects diplococci. What is the most likely diagnosis in this case?
A. Trichomoniasis
B. Syphilis
C Acute gonorrhea
D. Chlamydiosis
E. Candidiasis
Correct Answer: C. Acute gonorrhea
Explanation:
The patient presents with symptoms consistent with acute gonorrhea:
- Lower abdominal pain and fever are common in gonorrhea.
- Mucopurulent discharge from the genital tract is characteristic, especially with a burning sensation during urination.
- The physical findings of hyperemic and eroded cervix, painful uterus, and painful, thickened appendages suggest pelvic inflammatory disease (PID), which can result from untreated gonorrhea.
- The presence of diplococci on bacterioscopy is a key finding for Neisseria gonorrhoeae, the causative agent of gonorrhea.
Why Other Options Are Incorrect:
E. Candidiasis: Vaginal candidiasis is usually characterized by itching, thick white discharge, and vulvar irritation, not the mucopurulent discharge or the systemic symptoms (fever and pelvic pain) seen in this case.
A. Trichomoniasis: This infection is caused by a protozoan and typically causes vaginal discharge, irritation, and dysuria, but it does not lead to the same degree of pelvic pain and systemic symptoms (fever) seen in gonorrhea. The discharge in trichomoniasis is usually more frothy and greenish, not mucopurulent.
B. Syphilis: Syphilis typically begins with a painless ulcer (chancre) at the site of infection, not mucopurulent discharge or pelvic pain. It does not present with the acute inflammatory response described here.
D. Chlamydiosis: While chlamydia can cause similar symptoms, it often presents less acutely than gonorrhea and would not show diplococci on bacterioscopy. Chlamydia is also less likely to cause the intensity of pain and systemic symptoms like fever.